0% found this document useful (0 votes)
595 views136 pages

COMBO ALL ODISHA PREVIOUS YEAR QUESTION ANSWER OSSSC OSSC POLICE CT BED Other

Uploaded by

DILLEN KUMAR
Copyright
© © All Rights Reserved
We take content rights seriously. If you suspect this is your content, claim it here.
Available Formats
Download as PDF, TXT or read online on Scribd
0% found this document useful (0 votes)
595 views136 pages

COMBO ALL ODISHA PREVIOUS YEAR QUESTION ANSWER OSSSC OSSC POLICE CT BED Other

Uploaded by

DILLEN KUMAR
Copyright
© © All Rights Reserved
We take content rights seriously. If you suspect this is your content, claim it here.
Available Formats
Download as PDF, TXT or read online on Scribd
You are on page 1/ 136

ଓଡ଼ିଶାର ସମ� ପୂବର୍ ବଷର୍ର �ଶ� ଉ�ର ପାଇଁ

WtsApp/Call- 8596976190
Download “Tech Of World” App

ALL ODISHA 41,000 COMBO PYQ


E- Book (PDF)
All Odisha Exam Previous Year Chapter Wise
Question
[Exam- OSSSC, OSSC, OPSC, Police SI / Constable, Battalion,
Fireman, Jail Warder, B.ED, RHT, CT, OSSTET, OAVS, OTET,
OSSSC (PEO, RI, ARI, JA, LSI, OFDC, Group C)]

Tech Of World APP

Features Of “All” Chapter Wise COMBO PYQ

Subjects PYQ PYQ TEST


English 5,532
Math 5,565
Odia 4,409 184
Computer 2,577
Reasoning 4,181
Pedagogy 3,391
GK 15,667
Grand Total 41,322
Download App Click Here
To Buy
Contact / 8596976190
WhtsAp

Join Us-
ଓଡ଼ିଶାର ସମ� ପୂବର୍ ବଷର୍ର �ଶ� ଉ�ର ପାଇଁ
WtsApp/Call- 8596976190
Download “Tech Of World” App

All These Questions Have Been Updated To This E-Book

OSSSC Previous Year Questions (PYQ)


Sl. No. Exam Name Year
1 OSSSC RI 2015, 2021
2 OSSSC PEO & JA 2023
3 OSSSC CRE (ARI, AMIN, SFS, FG, EXCISE 2022
CONSTABLE)
4 OSSSC JUNIOR CLERK 2015, 2017, 2018
5 OSSSC LSI 2021
6 OSSSC ICDS Supervisor 2016
7 OSSSC Excise Constable 2014, 2019
8 OSSSC VAW 2016
9 OSSSC Jr. Stenographer 2015
10 OSSSC Laboratory Technician 2021
11 OSSSC MPHW 2023
12 OSSSC Nursing 2020, 2023
13 OSSSC SFS 2016
14 OSSSC Pharmacist 2020
15 OSSSC Radiographer 2020

Join Us-
ଓଡ଼ିଶାର ସମ� ପୂବର୍ ବଷର୍ର �ଶ� ଉ�ର ପାଇଁ
WtsApp/Call- 8596976190
Download “Tech Of World” App

OSSC Previous Year Questions (PYQ)


Sl. No. Exam Name Year
1 OSSC Assistant Sub-Inspector 2022
2 OSSC Accountant 2022
3 OSSC AMIN 2023 Pre & Mains
4 OSSC BSSO 2022 (All 18 Shifts)
5 OSSC CGL 2022 (All 18 Shifts) , 2023
6 OSSC CHSL 2023
7 OSSC CPGL 2023
8 OSSC CTS 2023
9 OSSC ESI 2023
10 OSSC GPEO 2015
11 OSSC Investigator 2022 (All 3 Shifts)
12 OSSC WEO 2022
13 OSSC JEA 2022 (All 16 Shifts)
14 OSSC Junior Assistant 2022 (All 10 Shifts)
15 OSSC Junior Clerk 2022
16 OSSC SCEW 2022 (All 5 Shifts)
17 OSSC SFS 2016
18 OSSC Specialist 2023
19 OSSC Stenographer 2023, 2024
20 OSSC Supply Inspector 2017
21 OSSC WEO 2022 (All 21 Shifts)

Join Us-
ଓଡ଼ିଶାର ସମ� ପୂବର୍ ବଷର୍ର �ଶ� ଉ�ର ପାଇଁ
WtsApp/Call- 8596976190
Download “Tech Of World” App

Odisha Police & Defence Previous Year Questions (PYQ)


Sl. No. Exam Name Year
1 Odisha Battalion / OSAP 2017
2 Odisha Police Constable 2011, 2012, 2013, 2018,
2023
3 Odisha Fireman 2023
4 Odisha Jail Warder 2022 (All 18 Shifts)
5 OPRB SI Police 2016, 2017, 2019, 2022
(All 11 Shifts), 2023
6 OSSC Excise SI 2021 (All 9 Shifts), 2022
7 OSSC Traffic Constable 2022 (All 6 Shifts)
8 OSSC Traffic SI 2022 (All 4 Shifts)

OPSC Previous Year Questions (PYQ)


Sl. No. Exam Name Year
1 OPSC ASO 2014, 2015, 2019, 2022
2 OPSC OMAS 2019, 2023
3 OPSC OAS Only Odisha GK Taken
From All Previous Year
Preliminary Exams
4 OPSC Assistant Director Of Fatory & Boiler 2023 Pre (Only Odisha GK
Taken)
5 OPSC Asst Director Handicraft 2023 Pre (Only Odisha GK
Taken)

Odisha High Court Previous Year Questions (PYQ)


Sl. No. Exam Name Year
1 OHC ASO 2021, 2023, 2023 (ST
Special)

Odisha Forest Development Corporation Previous Year Questions (PYQ)


Sl. No. Exam Name Year
1 OFDC Field Assistant 2021

Join Us-
ଓଡ଼ିଶାର ସମ� ପୂବର୍ ବଷର୍ର �ଶ� ଉ�ର ପାଇଁ
WtsApp/Call- 8596976190
Download “Tech Of World” App

Odisha Teaching Exams Previous Year Questions (PYQ)


Sl. No. Exam Name Year
1 CT 2018 (All 23 Shifts)
2019 (All 24 Shifts)
2020 (All 20 Shifts)
2023 (All 11 Shifts)
2 OTET 2022
3 JT 2023 (All 14 Shifts)
4 B.ED Entrance Exam
ARTS & SCIENCE 2018 (All 11 Shifts)
2019 (All 15 Shifts)
2020 (All 13 Shifts)
2021 (All 15 Shifts)
2022
2023
6 OSSTET (ARTS, CBZ & PCM) 2018, 2019, 2021 (1st),
2021 (2nd ), 2022, 2024
7 RHT / High School Teacher
 RHT ARTS 2019
2021 (All 3 Shifts)
2022 (All 6 Shifts)
2023 Pre (All 3 Shifts)
2023 Mains
 RHT CBZ 2019
2021
2022 (All 3 Shifts)
2023 Pre (All 3 Shifts)
2023 Mains
 RHT PCM 2019 (All 2 Shifts)
2021 (All 2 Shifts)
2022 (All 4 Shifts)
2023 Pre (All 3 Shifts)
2023 Mains
8 OAVS
OAVS TGT ODIA 2018, 2019, 2021
OAVS TGT ENGLISH 2019
OAVS TGT SOCIAL STUDIES 2019, 2021, 2023
OAVS TGT SCIENCE 2019, 2021
OAVS TGT MATH 2019, 2023

Join Us-
ଓଡ଼ିଶାର ସମ� ପୂବର୍ ବଷର୍ର �ଶ� ଉ�ର ପାଇଁ
WtsApp/Call- 8596976190
Download “Tech Of World” App

Click Here

Join Us-
ଓଡ଼ିଶାର ସମ� ପୂବର୍ ବଷର୍ର �ଶ� ଉ�ର ପାଇଁ
WtsApp/Call- 8596976190
Download “Tech Of World” App

ଓଡ଼ିଆ ବ୍ୟାକରଣ- ALL PYQ


PYQ E- Book & TEST
All Odisha Exam Previous Year Chapter Wise Question
[Exam- OSSSC, OSSC, OPSC, POLICE, B.ED, JT, RHT, CT, OAVS,
OSSTET, OTET, OSSSC (PEO, RI, ARI & Other) & Other
Tech Of World APP

Features Of “Odia Grammar” Chapter Wise PYQ E-Book & PYQ TEST

Total PYQ 4.409


Total PYQ TEST 184
Total Topic 30
Best For All Odisha Exam- OSSSC, OSSC, OPSC,
POLICE, B.ED, JT, RHT, CT, OAVS, OSSTET,
OTET, OSSSC (PEO, RI, ARI & Other) &
Other
Download App To Click Here
Buy Full E-Book
Call/ WhatsApp 8596976190

Click Here

Join Us-
ଓଡ଼ିଶାର ସମ� ପୂବର୍ ବଷର୍ର �ଶ� ଉ�ର ପାଇଁ
WtsApp/Call- 8596976190
Download “Tech Of World” App

Sl. No. Topic No. Of PYQ No. Of


PYQ TEST
1 ବିେଶଷ�, ବିେଶଷଣ, ସବ�ନାମ 318 17
2 ଶ� (ଶୁ� ଶ�, ଅଶୁ� ଶ�) 232 11
3 �ତି ଶ� 269 15
4 ବିପରୀତ ଶ� 203 11
5 ରୁଢ଼ି ଏବଂ େଲାକବାଣୀ 220 12
6 ଶ� (େଦଶଜ ,େବ�େଦଶିକ) 121 7
7 ଗଦ� 698
8 Translation 428 21
9 �ିୟା, କାଳ 151 8
10 କବିତା, ଗ�, ଉପନ�ାସ, ରଚନା, କବି, େଲଖକ ଏବଂ 144
ରଚୟି ତା
11 ସମାସ 119 6
12 ପଦ� 94
13 �ତ�ୟ 132 7
14 ଅବ�ୟ 62 3
15 ବିରାମ ଚି� 17 1
16 ଧାତୁ ଏବଂ ଉପସଗ� 134 7
17 ବାକ� ଓ ବାକ� ର ପୂ�� ବିନ�ାସ 143 7
18 ସ�ି ବିେ�ଦ 109 5
19 Miscellaneous 175 8
20 ବିଭ�ି 129 10
21 �ନି ଏବଂ ବ��ମାଳା 161 9
22 ଶ� (Mix) 75 4
23 ଛ� ଏବଂ ଅଳ�ାର 35 2
24 କାରକ 57 3
25 କ��ା ଏବଂ କମ� 13 1
26 ଲି � 30 2
27 ଏକ ପଦେର �କାଶ 65 3
28 ଶୂନ� �ାନ ପୂରଣ କର 47 2
29 ବଚନ 11 1
30 ଭି � ଶ� 17 1
Total 4,409 184

Join Us-
ଓଡ଼ିଶାର ସମ� ପୂବର୍ ବଷର୍ର �ଶ� ଉ�ର ପାଇଁ
WtsApp/Call- 8596976190
Download “Tech Of World” App

Topic- ଶ� (ଶୁ� & ଅଶୁ�)


1- ଠିକ ଶ�ଟି ଚି�ଟ କର ।

[OPSC ASO 2022]

[OSSTET 2022]

[Odisha B.Ed Entrance 2015]

[OSSC JEA 2022]

[OSSTET 2021 2nd]

(A) ଓତଃେପାତ

(B) ଓତଃେ�ାତ

(C) ଓତେପ�ାତ

(D) ଓତେ�ାତ

Ans- D

Join Us-
ଓଡ଼ିଶାର ସମ� ପୂବର୍ ବଷର୍ର �ଶ� ଉ�ର ପାଇଁ
WtsApp/Call- 8596976190
Download “Tech Of World” App

2- ଠିକ୍ ଶ�(ଉ�ର) ଚି�ଟ କର ।

[ASO- OPSC 2019]

[OSSC CGL 2022]

[OPSC ASO 2022]

[OSSC JEA 2022]

[OSSSC RI 2015]

(A) ଅନିବା� ଯ��

(B) ଅନୀବ�ାଯ��

(C) ଅନିବାଯ��

(D) ଅନୀବାଯ��

Ans- C

3- ଠିକ ଶ�ଟି ଚି�ଟ କର ।

[OPSC ASO 2022]

[Odisha B.Ed Entrance 2014]

[OSSC CGL 2022]

[OSSSC VAW 2016]

(A) ଦୂ ରବ�ା

(B) ଦୂ ରାବ�ା

(C) ଦୁ ରବ�ା

(D) ଦୁ ରାବ�ା

Ans- C

Join Us-
ଓଡ଼ିଶାର ସମ� ପୂବର୍ ବଷର୍ର �ଶ� ଉ�ର ପାଇଁ
WtsApp/Call- 8596976190
Download “Tech Of World” App

4- ଠିକ୍ ଶ�ଟି େକଉଟିଁ ବାଛ

[Odisha B.Ed Entrance 2017]

[OSSTET 2019]

[OPSC ASO 2022]

[Odisha Fireman 2023]

(a) ପୂଜ��ଦ

(b) ପୂଜ�ା�ଦ

(c) ପୂେଜା�ଦ

(d) ପୂଜା�ଦ

Ans- D

5- େକଉଟିଁ ଠିକ୍ ଶ� ?

[Odisha B.Ed Entrance 2015]

[OSSC SCEW 2022]

[OSSC Excise Police SI 2021]

[OSSC JEA 2022]

(a) ଯାନବାହାନ

(b) ଯାନବାହନ

(c) ଜାନବାହାନ

(d) ଜାନବାହନ

Ans- B

Join Us-
ଓଡ଼ିଶାର ସମ� ପୂବର୍ ବଷର୍ର �ଶ� ଉ�ର ପାଇଁ
WtsApp/Call- 8596976190
Download “Tech Of World” App

6- �ଦ� ଶ� ମ�ରୁ େକଉଟିଁ ଠିକ - ଚି�ାଇ ଦିଅ ?

[OSSC CGL 2022]

[OSSC Excise Police SI 2021]

[OSSC SCEW 2022]

(A) ଶଷି ଭୂଷଣ

(B) ଶଶିଭୁଷଣ

(C) ଶଶୀଭୂଷଣ

(D) ଶଶିଭୂଷଣ

Ans- D

7- େକଉଟିଁ ସଠିକ୍ ଶ� ନିରୂପଣ କର।

[Odisha CT Entrance 2018]

[OPSC ASO 2022]

[OSSC Traffic Constable 2022]

A) ସାବ�ଜନୀନ

B) ସବ�ଜନୀନ

C) ସାବ�ଜନିନ

D) ସବ�ଜନିନ

Ans- A

Join Us-
ଓଡ଼ିଶାର ସମ� ପୂବର୍ ବଷର୍ର �ଶ� ଉ�ର ପାଇଁ
WtsApp/Call- 8596976190
Download “Tech Of World” App

8- ଶୁ� ଶ� ଦୁ ଇଟିକୁ ବାଛ |

[OSSSC Combined Recruitment 2022]

[Odisha B.Ed Entrance 2015]

[OSSTET 2019]

(A) ଶିକାର, ଶି�ୟ�ୀ

(B) ଶିକାର, ଶି�ୟି �ୀ

(C) ଶୀକାର, ଶି�ୟ�ୀ

(D) ଶୀକାର, ଶି�ୟି �ୀ

Ans- B

9- �ଦ� ଶ� ମ�ରୁ େକଉ ଁ ଶ�ଟି ଠିକ - ଚି�ାଇ ଦିଅ ?

[Odisha Police ASI 2022]

[OPSC ASO 2022]

[Odisha District Police Constable 2013]

(A) ଆ�ିବାଦ

(B) ଆଶିବା� ଦ

(C) ଆ�ୀବାଦ

(D) ଆଶୀବ�ାଦ

Ans- D

Join Us-
ଓଡ଼ିଶାର ସମ� ପୂବର୍ ବଷର୍ର �ଶ� ଉ�ର ପାଇଁ
WtsApp/Call- 8596976190
Download “Tech Of World” App

10- ଠିକ ଶ�ଟି ଚି�ାଇ ଦିଅ।

[OSSC Excise Police SI 2021]

[OSSC JEA 2022]

(A) ଜାତ�ାଭୀମାନ

(B) ଜାତ�ଭି ମାନୀ

(C) ଜାତ�ାଭି ମାନୀ

(D) ଯାତ�ାଭି ମାନ

Ans- B

To Continue - Click here

Join Us-
ଓଡ଼ିଶାର ସମ� ପୂବର୍ ବଷର୍ର �ଶ� ଉ�ର ପାଇଁ WtsApp/Call- 8596976190
Download “Tech Of World” App

ALL ODISHA EXAM GK PYQ


E- Book (PDF)
All Odisha Exam Previous Year Chapter Wise GK
Question & Answer With EXPLANATION
[Exam- OSSSC, OSSC, OPSC, Police SI /Constable, Battalion, Jail
Warder, B.ED, RHT, CT, OSSTET, OAVS, OTET (RI, ARI, JA, LSI,
OFDC, Group C)]

Tech Of World APP

Features Of “GK” Chapter Wise PYQ E-Book


Total PYQ 15,667+

Total Topic 95

Language Odia & English

Explanation Odia & English Language

Best For All Odisha Exam- OSSSC , OSSC , OPSC, Police SI


/Constable, Jail Warder, Battalion, Fireman, B.ED, JT,
RHT, CT, OAVS, OSSTET, OTET & Other

Download App To Buy Click Here


Full E-Book

Call/ WhatsApp 8596976190

Click Here

Join Us-
ଓଡ଼ିଶାର ସମ� ପୂବର୍ ବଷର୍ର �ଶ� ଉ�ର ପାଇଁ WtsApp/Call- 8596976190
Download “Tech Of World” App

Geography No. Of MCQ

1 Census 31

2 Climate & Weather 72

3 Crops & Vegetation 77

4 Lake 79

5 Environment & Ecology 100

6 Forest 86

7 Longitude & Latitude 166

8 Miscellaneous 538

9 Mountains 199

10 National Park & Other Projects 140

11 Resource Of Energy 50

12 Rivers 372

13 Rock & Minerals 112

14 Sea, Ocean, Coastal Area 88

15 Soil 62

16 The Earth & the Universe 241

17 Time Zone 48

18 Dam & Port 19

19 MAP 23

20 Physiography 10

21 Producer & Production 12

Total 2,525

Join Us-
ଓଡ଼ିଶାର ସମ� ପୂବର୍ ବଷର୍ର �ଶ� ଉ�ର ପାଇଁ WtsApp/Call- 8596976190
Download “Tech Of World” App

History No. Of MCQ


1 Ancient India 401

2 Medieval India 264

3 Modern History 1157

4 World History 130

5 National Congrees 45

Total 1,997

Polity No. Of MCQ

1 Act & Amendments 71

2 Article 243

3 Fundamental Rights & Duties 144

4 Judiciary (Supreme & High Court ) 161

5 Miscellaneous 193

6 Panchayat Raj System 51

7 Parliament, Lok , Rajya, Bidhana Sabha 404

8 Preamble, Parts and Schedules of 490


Constitution
9 President & Governor 162

10 Election 38

Total 1,957

Join Us-
ଓଡ଼ିଶାର ସମ� ପୂବର୍ ବଷର୍ର �ଶ� ଉ�ର ପାଇଁ WtsApp/Call- 8596976190
Download “Tech Of World” App

Statics GK No. Of PYQ

1 Books 99

2 Culture, Festival, Dance & Language 108

3 First, Last, Largest, Smallest, Longest, 208


Shortest
4 Important Dates 91

5 Rank, Index, Report 62

6 Invention & Discovery 23

7 Miscellaneous 452

8 Capital, Headquarter, Organizations & 377


Institutions
9 Prizes & Awards 98

10 Sport 134

11 Fact Of Indian State 76

12 Full Form 33

13 Science & Technology 10

Total 1,771

Odisha GK No. Of PYQ

1 Odisha Geography & Odisha Static GK 452

2 Odisha History 252

Total 704

Join Us-
ଓଡ଼ିଶାର ସମ� ପୂବର୍ ବଷର୍ର �ଶ� ଉ�ର ପାଇଁ WtsApp/Call- 8596976190
Download “Tech Of World” App

Biology No. Of PYQ

1 Cell Division 40

2 Cell- Structure & Function 252

3 Diseases 144

4 Diversity In Living World 208

5 Ecology & Ecosystem 209

6 Genetics 60

7 Human Physiology 280

8 Micro Organism 95

9 Miscellaneous 60

10 Nutrition 28

11 Plant Morphology Physiology 424

12 Reproduction System 54

13 Tissue 21

14 Vitamin 33

Total 1,968

Join Us-
ଓଡ଼ିଶାର ସମ� ପୂବର୍ ବଷର୍ର �ଶ� ଉ�ର ପାଇଁ WtsApp/Call- 8596976190
Download “Tech Of World” App

Chemistry No. Of PYQ

1 Acid Base & Salt 164

2 Atmosphere 47

3 Carbon & Its Compound 29

4 Chemical Bond 50

5 Chemical Reaction & Equation 265

6 Chemistry In Every Day Life 76

7 Electro Chemistry 29

8 Extraction Of Solid 23

9 Gaseous State 68

10 Hydro-Carbon Compound 135

11 Metal & Non-Metal 114

12 Miscellaneous 43

13 Periodic Table 85

14 Solution And Colloids 29

15 State Of Matter 135

16 Structure Of Atom 222

Total 1,514

Join Us-
ଓଡ଼ିଶାର ସମ� ପୂବର୍ ବଷର୍ର �ଶ� ଉ�ର ପାଇଁ WtsApp/Call- 8596976190
Download “Tech Of World” App

Physics No. Of PYQ

1 Electricity, Current & Magnetic Field 162

2 Force, Work, Energy, Power 203

3 Heat & Temperature (Thermodynamics) 54

4 Light 110

5 Magnet 21

6 Miscellaneous 228

7 Motion (Distance, Velocity & Accelaration) 213

8 Physics New PYQ 65

9 Renewable Energy 6

10 Sound 64

11 Unit & Formula 89

12 Wave 50

Total 1,265

Other GK No. Of PYQ


1 Current Affairs 1413

2 Economics 553

Join Us-
ଓଡ଼ିଶାର ସମ� ପୂବର୍ ବଷର୍ର �ଶ� ଉ�ର ପାଇଁ WtsApp/Call- 8596976190
Download “Tech Of World” App

All Subject No. Of PYQ

1 Geography 2525
2 History 1997
3
Polity 1957

4 Statics GK 1771
5 Odisha GK 704
6 Biology 1968
7 Chemistry 1514
8 Physics 1265
9 Current Affairs 1413
10 Economics 553

Grand Total 15,667

Join Us-
ଓଡ଼ିଶାର ସମ� ପୂବର୍ ବଷର୍ର �ଶ� ଉ�ର ପାଇଁ WtsApp/Call- 8596976190
Download “Tech Of World” App

Topic Name- Odisha Geography

1- Select the States through which the river Mahanadi flows.

େଯଉଁ ରାଜ� େଦଇ ମହାନଦୀ �ବାହିତ େହଉଛି ତାହା ଚୟନ କର�

[A] Odisha and Jharkhand

[B] Odisha and Chhattisgarh

[C] Odisha, Jharkhand and Chhattisgarh

[D] Odisha, Telangana and Chhattisgarh

[Odisha District Police Constable 2023]

Ans- B

The Mahanadi is a major river in India that flows through the states of
Chhattisgarh and Odisha. It is one of the longest rivers in India, with a
total length of approximately 858 kilometers. The river originates in the
Dhamtari district of Chhattisgarh and flows in a southeast direction
before entering Odisha. In Odisha, the Mahanadi passes through several
districts, including Sambalpur, Cuttack, and Puri, before finally
emptying into the Bay of Bengal near Paradip.

ମହାନଦୀ ଭାରତର ଏକ �ମୁଖ ନଦୀ ଯାହା ଛତିଶଗଡ଼ ଓ ଓଡ଼ିଶା ରାଜ� େଦଇ �ବାହିତ
େହାଇଥାଏ । ଏହା ଭାରତର ସବୁ ଠାରୁ ଲ��ା ନଦୀ ମ�ରୁ ଅନ�ତମ, ଯାହାର େମାଟ ଲ�� �ାୟ
୮୫୮ କିେଲାମିଟର ଅେଟ । ଏହି ନଦୀ ଛତିଶଗଡ଼ର ଧମତାରୀ ଜି�ାରୁ ବାହାରି ଦ�ିଣ-ପୂବ �
ଦିଗେର �ବାହିତ େହାଇ ଓଡ଼ିଶାେର �େବଶ କରି ଛି। ଓଡ଼ିଶାର ସ��ଲପୁର, କଟକ ଓ ପୁରୀ
ସେମତ ଅେନକ ଜି�ା େଦଇ ମହାନଦୀ �ବାହିତ େହାଇ େଶଷେର ପାରାଦୀପ ନିକଟବ��ୀ
ି ି।
ବେ�ାପସାଗରେର ମିଶଛ

Join Us-
ଓଡ଼ିଶାର ସମ� ପୂବର୍ ବଷର୍ର �ଶ� ଉ�ର ପାଇଁ WtsApp/Call- 8596976190
Download “Tech Of World” App

2- Which type of climate does Odisha enjoy?

- ଓଡ଼ିଶାର େକଉ ଁ �କାର ଜଳବାୟୁ ରହିଛ?


ି

(A) Tropical

(B) Temperate

(C) Equtorial

(D) Tundra

[OSSSC RI Exam 2021]

Ans- A

Odisha enjoys a tropical climate. The state experiences hot and humid
weather for most of the year. Summers are typically hot with
temperatures reaching high levels, while winters are mild and pleasant.
The state also receives a significant amount of rainfall during the
monsoon season, which extends from June to September. The tropical
climate of Odisha supports the growth of diverse flora and fauna in the
region.

ଓଡ଼ିଶାେର �ୀ��ମ�ଳୀୟ ଜଳବାୟୁ ରହିଛି। ରାଜ�େର ବଷ�ର ଅଧ �କାଂଶ ସମୟେର ଗରମ ଓ


ଆ�� ପାଗ ଅନୁ ଭୂତ େହାଇଥାଏ। �ୀ��ଋତୁ ସାଧାରଣତଃ ଗରମ େହବା ସହ ତାପମା�ା ଉ�
�ରେର ପହ�ିଥାଏ ଏବଂ ଶୀତ ସାମାନ� ଏବଂ ଆନ�ଦାୟକ େହାଇଥାଏ । ଜୁନ୍- େସେ���ର

ପଯ��� ଚାଲି ଥ �ବା େମୗସୁମୀ ଋତୁ େର ମ� ରାଜ�େର ଯେଥ� ପରି ମାଣର ବଷ�ା େହାଇଥାଏ।
ଓଡ଼ିଶାର �ୀ��ମ�ଳୀୟ ଜଳବାୟୁ ଏହି ଅ�ଳେର ବିଭି� ଉ�ି ଦ ଓ ଜୀବଜ�� ବିକାଶକୁ
ସମଥ�ନ କରି ଥାଏ ।

Join Us-
ଓଡ଼ିଶାର ସମ� ପୂବର୍ ବଷର୍ର �ଶ� ଉ�ର ପାଇଁ WtsApp/Call- 8596976190
Download “Tech Of World” App

3- Which state is to the West of Odisha?

ଓଡ଼ିଶାର ପ�ି ମେର େକଉ ଁ ରାଜ� ଅଛି?

A. Jharkhand

B. Chhattisgarh

C. Goa

D. None of these

[Odisha Jail Warder 2022]

Ans- B

1. West Bengal: Odisha shares its border with the state of West
Bengal to the northeast.
2. Jharkhand: To the north and northwest of Odisha, it is bordered
by the state of Jharkhand.
3. Chhattisgarh: Odisha shares its western border with the state of
Chhattisgarh.
4. Andhra Pradesh: To the south of Odisha, it is bordered by the
state of Andhra Pradesh.

1. ପ�ି ମବ�: ଉ�ର-ପୂବେ� ର ପ�ି ମବ� ରାଜ� ସହ ଓଡ଼ିଶାର ସୀମା ରହିଛି।


2. ଝାଡ଼ଖ�: ଓଡ଼ିଶାର ଉ�ର ଓ ଉ�ର-ପ�ି ମ ଦିଗେର ଝାଡ଼ଖ� ରାଜ�ର
ସୀମା�େର ଅବ�ିତ।
3. ଛତିଶଗଡ଼: ଓଡ଼ିଶାର ପ�ି ମ ସୀମା ଛତିଶଗଡ଼ ରାଜ� ସହ ରହିଛି।
4. ଆ��େଦଶ: ଓଡ଼ିଶାର ଦ�ିଣଦିଗେର ଆ��େଦଶ ରାଜ୍ୟ ସୀମା�େର ଅବ�ିତ।

Join Us-
ଓଡ଼ିଶାର ସମ� ପୂବର୍ ବଷର୍ର �ଶ� ଉ�ର ପାଇଁ WtsApp/Call- 8596976190
Download “Tech Of World” App

4. Budhabalanga River flows through which districts of Odisha?

. ବୁ ଢ଼ାବଳ�ା ନଦୀ ଓଡ଼ିଶାର େକଉ ଁ ଜି�ା େଦଇ �ବାହିତ େହଉଛି?

A) Mayurbhanj and Balasore

B) Koraput and Nabarangpur

C) Nayagarh and Puri

D) Bolangir and Bargarh

[Odisha Battalion 2017]

Ans- A

It originates from the Similipal Hills in the Mayurbhanj district of


Odisha and flows in a northeast direction. It flows in Mayurbhanj district
and Balasore district and finally empties into the Bay of Bengal near
Balaramgadi village of Balasore district. The river travels a total length
of 175 Km.

ଏହା ଓଡ଼ିଶାର ମୟୂରଭ� ଜି�ାର ଶିମିଳିପାଳ ପାହାଡ଼ରୁ ବାହାରି ଉ�ର-ପୂବ � ଦିଗେର �ବାହିତ
େହଉଛି। ଏହା ମୟୂରଭ� ଜି�ା ଓ ବାେଲ�ର ଜି�ାେର �ବାହିତ େହାଇ େଶଷେର ବାେଲ�ର
ଜି�ାର ବଳରାମଗଡ଼ି ଗାଁ ନିକଟେର ବେ�ାପସାଗରେର ମିଶଛ
ି ି। ଏହି ନଦୀର ସମୁଦାୟ େଦ�ଘ��
୧୭୫ କିେଲାମିଟର ।

Join Us-
ଓଡ଼ିଶାର ସମ� ପୂବର୍ ବଷର୍ର �ଶ� ଉ�ର ପାଇଁ WtsApp/Call- 8596976190
Download “Tech Of World” App

5- Which of the following is the most Urbanised district of Odisha


as per the 2011 Census?

- 2011 ଜନଗଣନା ଅନୁ ଯାୟୀ ନି�ଲ


� ି ଖତ
� ମ�ରୁ େକଉଟିଁ ଓଡ଼ିଶାର ସବ�ାଧ�କ ନଗରୀକୃତ
ଜି�ା ଅେଟ?

(A) Khordha

(B) Angul

(C) Jharsuguda

(D) Kandhamal

[OSSC BSSO 2022]

Ans- A

According to the 2011 Census, the most urbanized district in Odisha is


Khordha.

1. Population: The total population of Odisha in 2011 was


approximately 41.9 million, making it the 11th most populous
state in India.
2. Literacy Rate: The literacy rate in Odisha was 72.87% in 2011,
reflecting the percentage of the population aged seven years and
above who can read and write with understanding.
3. Sex Ratio: The sex ratio in Odisha was 978 females per 1000
males in 2011, indicating a slight imbalance in favor of males.
4. Urbanization: The urban population of Odisha was 16.59 million,
accounting for about 16% of the total population of the state.
5. Scheduled Tribes: Odisha has a significant tribal population. In
2011, the state had a total of 62 recognized Scheduled Tribes.

Join Us-
ଓଡ଼ିଶାର ସମ� ପୂବର୍ ବଷର୍ର �ଶ� ଉ�ର ପାଇଁ WtsApp/Call- 8596976190
Download “Tech Of World” App

୨୦୧୧ ଜନଗଣନା ଅନୁ ଯାୟୀ, ଓଡ଼ିଶାର ସବୁ ଠାରୁ ଅଧ �କ ନଗରୀକରଣ ଜି�ା େହଉଛି
େଖା��ା ।

1. ଜନସଂଖ�ା: ୨୦୧୧ ମସିହାେର ଓଡ଼ିଶାର େମାଟ ଜନସଂଖ�ା �ାୟ ୪୧.୯ ନିୟୁତ ଥ �ଲା,
ଯାହା ଭାରତର ୧୧ତମ ସବ�ାଧ �କ ଜନବହୁ ଳ ରାଜ� ଥ �ଲା ।
2. ସା�ରତା ହାର: ୨୦୧୧ ମସି ହାେର ଓଡ଼ିଶାେର ସା�ରତା ହାର ୭୨.୮୭% ଥ �ଲା, ଯାହା
ସାତ ବଷ� କି�ା� ତା'ଠାରୁ ଅଧ �କ ବୟସର ଜନସଂଖ�ାର �ତିଶତକୁ �ତିଫଳିତ କେର
େଯଉମାେନ
ଁ ବୁ ଝାମଣା ସହିତ ପଢ଼ିବା ଏବଂ େଲଖ �ପାରି େବ ।
3. ଲି � ଅନୁ ପାତ: ୨୦୧୧େର ଓଡ଼ିଶାେର ଲି � ଅନୁ ପାତ �ତି ୧୦୦୦ ପୁରୁଷେର ୯୭୮ ମହିଳା
ଥ �ଲା, ଯାହା ପୁରୁଷ� ସପ�େର ସାମାନ� ଅସ�ଳନକୁ ସୂଚାଉଛି।
4. ସହରୀକରଣ: ଓଡ଼ିଶାର ସହରା�ଳ ଜନସଂଖ�ା ୧୬.୫୯ ନିୟୁତ ଥ �ଲା, ଯାହା ରାଜ୍ୟର େମାଟ
ଜନସଂଖ�ାର �ାୟ ୧୬% ଅେଟ ।
5. ଅନୁ ସଚ
ୂ ିତ ଜନଜାତି: ଓଡ଼ିଶାେର ବହୁ ସଂଖ�ାେର ଆଦିବାସୀ ଅଛ�ି। ୨୦୧୧ମସିହାେର
ରାଜ�େର େମାଟ ୬୨ଟି �ୀକୃତି�ା� ଅନୁ ସଚ
ୂ ିତ ଜନଜାତି ଥ �େଲ।

6- Which district of Odisha is the largest producer of rice?

- ଓଡ଼ିଶାର େକଉ ଁ ଜି�ା ସବୁଠାରୁ ଅଧ�କ ଚାଉଳ ଉ�ାଦନକାରୀ ଜି�ା?

(A) Koraput

(B) Bolangir

(C) Bargarh

(D) Cuttack

[OSSSC Livestock Inspector 2021]

Ans- C

Join Us-
ଓଡ଼ିଶାର ସମ� ପୂବର୍ ବଷର୍ର �ଶ� ଉ�ର ପାଇଁ WtsApp/Call- 8596976190
Download “Tech Of World” App

Bargarh district produced 2.2 million tonnes of rice in 2020-21,


accounting for about 30% of the total rice production in Odisha. The
Attabira block in Bargarh district is known as the "Rice Bowl Of Odisha"
due to its high productivity of rice.

୨୦୨୦-୨୧େର ବରଗଡ଼ ଜି�ାେର ୨୨ ଲ� ଟନ୍ ଚାଉଳ ଉ�ାଦନ େହାଇଥ �ଲା, ଯାହା ଓଡ଼ିଶାର
େମାଟ ଚାଉଳ ଉ�ାଦନର �ାୟ ୩୦ �ତିଶତ। ବରଗଡ଼ ଜି�ାର ଅତାବିରା ��କେର ଚାଉଳର
ଉ�ାଦକତା ଅଧ �କ ଥ �ବାରୁ ଏହାକୁ “ଓଡ଼ିଶାର ଚାଉଳ ବାଟି” େବାଲି କୁହାଯାଏ।

7- Which of the following is the highest waterfall in Odisha?

- ନି�ଲ
� ି ଖତ
� ମ�ରୁ େକଉଟିଁ ଓଡ଼ିଶାର ସବ�ାଧ�କ ଜଳ�ପାତ ?

(A) Barehipani Waterfalls

(B) Sanaghagara Waterfalls

(C) Kalhatti Waterfalls

(D) Hatipathar Waterfalls

[OSSC WEO Exam 2022]

Ans- A

The answer is (A). Barehipani Waterfalls is the highest waterfall in


Odisha, with a height of 400 meters (1,312 feet). It is located in the
Simlipal National Park in Mayurbhanj district.

• Sanaghagara Waterfalls (300 meters / 984 feet) - Located in the


Deogarh district.
• Kalhatti Waterfalls (250 meters / 820 feet) - Located in the
Sundargarh district.
• Hatipathar Waterfalls (150 meters / 492 feet) - Located in the
Rayagada district.

Join Us-
ଓଡ଼ିଶାର ସମ� ପୂବର୍ ବଷର୍ର �ଶ� ଉ�ର ପାଇଁ WtsApp/Call- 8596976190
Download “Tech Of World” App

ଏହାର ଉ�ର େହଉଛି (ଏ)। ୪୦୦ ମିଟର (୧୩୧୨ ଫୁ ଟ) ଉ�ତା ବିଶ�


ି ବେରହିପାଣି
ଜଳ�ପାତ ଓଡ଼ିଶାର ସେବ�ା� ଜଳ�ପାତ । ଏହା ମୟୂରଭ� ଜି�ାର ଶିମିଳିପାଳ ଜାତୀୟ
ଉଦ�ାନେର ଅବ�ିତ ।

• ସାନଘାଗରା ଜଳ�ପାତ (୩୦୦ ମିଟର/ ୯୮୪ ଫୁ ଟ) - େଦବଗଡ଼ ଜି�ାେର ଅବ�ିତ ।


• କଲହଟି ଜଳ�ପାତ (୨୫୦ ମିଟର/ ୮୨୦ ଫୁ ଟ) - ସୁ�ରଗଡ଼ ଜି�ାେର ଅବ�ିତ ।
• ହାତୀପଥର ଜଳ�ପାତ (୧୫୦ ମିଟର/ ୪୯୨ ଫୁ ଟ) - ରାୟଗଡ଼ା ଜି�ାେର ଅବ�ିତ ।

8- Originating from Chhotanagpur plateau, the Brahmani enters


Odisha in which district '?

- େଛାଟନାଗପୁର ମାଳଭୂମିରୁ �ା�ଣୀ େକଉ ଁଜି�ାେର ଓଡ଼ିଶାେର �େବଶ କରି ଛ'ି ?

(A) Sundargarh

(B) Bargarh

(C) Keonjhar

(D) Sambalpur

[OSSSC Combined Recruitment 2022]

Ans- A

Join Us-
ଓଡ଼ିଶାର ସମ� ପୂବର୍ ବଷର୍ର �ଶ� ଉ�ର ପାଇଁ WtsApp/Call- 8596976190
Download “Tech Of World” App

The Brahmani River originates from the Chhotanagpur Plateau in


Jharkhand and enters Odisha in the Sundargarh district. It is the
second-longest river in Odisha after the Mahanadi River. The
Brahmani River flows through the districts of Sundargarh, Deogarh,
Angul, Dhenkanal, Cuttack, Jajpur, and Kendrapara before finally
emptying into the Bay of Bengal.

Some of the important waterfalls on the Brahmani River:-

• Joranda Falls - Located in the Sundargarh district


• Rengali Falls - Located in the Angul district
• Dhauli Falls - Located in the Cuttack district
• Hussain Sagar Falls - Located in the Kendrapara district

�ା�ଣୀ ନଦୀ ଝାଡ଼ଖ�ର େଛାଟନାଗପୁର ମାଳଭୂମିରୁ ବାହାରି ସୁ�ରଗଡ଼ ଜି�ାର


ଓଡ଼ିଶାେର �େବଶ କରି ଥାଏ। ମହାନଦୀ ପେର ଏହା ଓଡ଼ିଶାର �ିତୀୟ ଦୀଘ�ତମ ନଦୀ
। ସୁ�ରଗଡ଼, େଦବଗଡ଼, ଅନୁ ଗଳ
ୁ , େଢ�ାନାଳ, କଟକ, ଯାଜପୁର ଓ େକ�ାପଡ଼ା ଜି�ା
ି ।ି
େଦଇ �ା�ଣୀ ନଦୀ �ବାହିତ େହାଇ େଶଷେର ବେ�ାପସାଗରେର ମିଶଛ

�ା�ଣୀ ନଦୀର େକେତକ ଗୁରୁ�ପୂ�� ଜଳ�ପାତ:-

• େଯାର�ା ଜଳ�ପାତ - ସୁ�ରଗଡ଼ ଜି�ାେର ଅବ�ିତ


• େର�ାଲି ଜଳ�ପାତ - ଅନୁ ଗଳ
ୁ ଜି�ାେର ଅବ�ିତ
• ଧଉଳି ଜଳ�ପାତ - କଟକ ଜି�ାେର ଅବ�ିତ
• ହୁ େସନ ସାଗର ଜଳ�ପାତ - େକ�ାପଡ଼ା ଜି�ାେର ଅବ�ିତ

Join Us-
ଓଡ଼ିଶାର ସମ� ପୂବର୍ ବଷର୍ର �ଶ� ଉ�ର ପାଇଁ WtsApp/Call- 8596976190
Download “Tech Of World” App

9- In which district of Odisha the Hot Spring Atri is located?

- ଓଡ଼ିଶାର େକଉ ଁ ଜି�ାେର ହଟ୍ �ି�ଂ ଅ�ୀ ଅବ�ିତ ?

(A) Nayagarh

(B) Khurda

(C) Ganjam

(D) Cuttack

[OSSC Excise SI]

Ans- B

Hot Spring Atri is located in the Khordha district of Odisha, about 15


kilometers west of the district headquarters town of Khurda. The spring is
a popular tourist destination and is known for its hot water, which is said
to have medicinal properties. The water temperature of the spring is
around 57 degrees Celsius (134 degrees Fahrenheit).

େଖା��ା ଜି�ା ସଦର ମହକୁମା ଠାରୁ �ାୟ ୧୫ କିେଲାମିଟର ପ�ି ମେର ଓଡ଼ିଶାର େଖା��ା
ଜି�ାେର ହଟ୍ �ି�ଂ ଅ�ୀ ଅବ�ିତ । ବସ� ଏକ େଲାକ�ି ୟ ପଯ��ଟନ�ଳୀ ଏବଂ ଏହାର ଗରମ
ପାଣି ପାଇ ଁ ଜଣାଶୁଣା, ଯାହାର ଔଷଧୀୟ ଗୁଣ ଅଛି େବାଲି କୁହାଯାଏ । ବସ� ଋତୁ ର ଜଳ
ତାପମା�ା ପାଖାପାଖ � ୫୭ ଡି�ୀ େସଲସି ୟସ (୧୩୪ ଡି�ୀ ଫାେରନହାଇଟ୍ ) ଅେଟ ।

Join Us-
ଓଡ଼ିଶାର ସମ� ପୂବର୍ ବଷର୍ର �ଶ� ଉ�ର ପାଇଁ WtsApp/Call- 8596976190
Download “Tech Of World” App

10- Which of the following is the highest peak in the state of


Odisha?

- ନି�ଲ
� ି ଖତ
� ମ�ରୁ େକଉଟିଁ ଓଡ଼ିଶା ରାଜ�ର ସେବ�ା� ଶିଖର ?

(A) Malayagiri

(B) Mahendragiri

(C) Deomali

(D) Turia Konda

[OSSC CGL Exam 2022]

Ans- C

The highest peak in Odisha is Deomali, which has an elevation of 1,672


meters (5,485 feet). It is located in the Koraput district of Odisha, and is
part of the Eastern Ghats mountain range.
Here are some other high peaks in Odisha :-

• Mahendragiri - 1,501 meters (4,925 feet)


• Turia Konda - 1,562 meters (5,123 feet)
• Sinkaram - 1,620 meters (5,314 feet)
• Golikoda - 1,617 meters (5,307 feet)
• Yendrika - 1,582 meters (5,187 feet)

Join Us-
ଓଡ଼ିଶାର ସମ� ପୂବର୍ ବଷର୍ର �ଶ� ଉ�ର ପାଇଁ WtsApp/Call- 8596976190
Download “Tech Of World” App

ଓଡ଼ିଶାର ସେବ�ା� ଶିଖର େହଉଛି େଦଓମାଳି, ଯାହାର ଉ�ତା ୧,୬୭୨ ମିଟର (୫,୪୮୫ ଫୁ ଟ)
ଅେଟ । ଏହା ଓଡ଼ିଶାର େକାରାପୁଟ ଜି�ାେର ଅବ�ିତ ଏବଂ ପୂବ � ଘାଟ ପବ�ତମାଳାର ଏକ ଅଂଶ
ଅେଟ ।

ଜାଣ� ଓଡ଼ିଶାର ଆଉ େକେତକ ଉ� ଶିଖର ()

• ମେହ�ଗିରି - ୧୫୦୧ ମିଟର (୪୯୨୫ ଫୁ ଟ)


• ତୁ ରି ଆ େକା�ା - ୧,୫୬୨ ମିଟର (୫,୧୨୩ ଫୁ ଟ)
• ସି �ରାମ - ୧,୬୨୦ ମିଟର (୫,୩୧୪ ଫୁ ଟ)
• େଗାଲି େକାଡା - ୧,୬୧୭ ମିଟର (୫,୩୦୭ ଫୁ ଟ)
• େୟ�ିକା - ୧,୫୮୨ ମିଟର (୫,୧୮୭ ଫୁ ଟ)

11- Why is Chilika Lake called a Lagoon?

- ଚିଲିକା �ଦକୁ କାହିଁକି ଲାଗୁନ୍ କୁହାଯାଏ?

(A) It is formed due to damming of sections of river channels.

(B) It is formed due to flowing in of rain water into local depressions.

(C) It is produced due to erosional and depositional action of waves in


the coastal regions.

(D) It is formed by remnants of ancient ocean and sea areas.

[OPSC ASO Exam 2019]

Ans- C

Join Us-
ଓଡ଼ିଶାର ସମ� ପୂବର୍ ବଷର୍ର �ଶ� ଉ�ର ପାଇଁ WtsApp/Call- 8596976190
Download “Tech Of World” App

• Chilika Lake is the largest brackish water lagoon in India and the
second largest in the world. It is located in the Puri, Khurda, and
Ganjam districts of Odisha.
• Chilika Lake is a Ramsar wetland of international importance. It
was declared a National Park in 1981 and a biosphere reserve
in 1989.
• The lake has a maximum length of 64.3 km (40.0 mi), a maximum
width of 25.6 km (15.9 mi), and a maximum depth of 4.2 m (13.8
ft).

• ଚିଲିକା �ଦ େହଉଛି ଭାରତର ସବୁ ଠାରୁ ବଡ଼ ଲୁ ଣା ଜଳ �ବାହ ଏବଂ ବି�ର �ିତୀୟ


ବୃ ହ�ମ �ଦ । ଏହା ଓଡ଼ିଶାର ପୁରୀ, େଖା��ା ଓ ଗ�ାମ ଜି�ାେର ଅବ�ିତ ।
• ଚିଲିକା �ଦ ଅ�ଜ�ାତୀୟ �ରେର ଗୁରୁ�ପୂ�� ଏକ ରାମସାର ଆ��ଭୂମି ।
୧୯୮୧ମସି ହାେର ଏହାକୁ ଜାତୀୟ ଉଦ�ାନ ଓ ୧୯୮୯ମସିହାେର େଜ�ବମ�ଳ ସଂର�ିତ
ଅ�ଳ ଭାେବ େଘାଷଣା କରାଯାଇଥ �ଲା।
• �ଦର ସବ�ାଧ �କ ଲ�� ୬୪.୩ କିେଲାମିଟର (୪୦.୦ ମାଇଲ), ସବ�ାଧ �କ �� ୨୫.୬
କିେଲାମିଟର (୧୫.୯ ମାଇଲ) ଏବଂ ସବ�ାଧ �କ ଗଭୀରତା ୪.୨ ମିଟର (୧୩.୮ ଫୁ ଟ)
ରହିଛି ।

12- Which of the following statement is/are TRUE?

- ନି�ଲ
� ି ଖତ
� ବିବୃ�ି ମ�ରୁ େକଉଟିଁ ସତ� ଅେଟ/ ଅେଟ?

A. The highest mountain peak in the state of Odisha is Deomali

B. Deomali is situated in Malkangiri district

A. ଓଡ଼ିଶା ରାଜ�ର ସେବ�ା� ପବ�ତ ଶିଖର େହଉଛି େଦଓମାଳି

B.େଦଓମାଳି ମାଲକାନଗି ରି ଜି�ାେର ଅବ�ିତ ।

A) Statement A is False and B is True

B) Both the statements A and B are False

C) Both the statements A and B are True

Join Us-
ଓଡ଼ିଶାର ସମ� ପୂବର୍ ବଷର୍ର �ଶ� ଉ�ର ପାଇଁ WtsApp/Call- 8596976190
Download “Tech Of World” App

D) Statement A is True and B is False

[OSSC Traffic Constable 2022 Exam]

Ans- D

The highest peak in Odisha is Deomali, It is located in the Koraput


district of Odisha, and is part of the Eastern Ghats mountain range.

ଓଡ଼ିଶାର ସେବ�ା� ଶିଖର େହଉଛି େଦଓମାଳି, ଏହା ଓଡ଼ିଶାର େକାରାପୁଟ ଜି�ାେର ଅବ�ିତ
ଏବଂ ପୂବ � ଘାଟ ପବ�ତମାଳାର ଏକ ଅଂଶ ଅେଟ ।

13- Satkosia Gorge wildlife sanctuary is situated in which of the


following districts of Odisha?

- ସାତେକାଶିଆ ବନ��ାଣୀ ଅଭୟାରଣ� ଓଡ଼ିଶାର ନି�ଲ


� ି ଖତ
� େକଉ ଁ ଜି�ାେର ଅବ�ିତ ?

(A) Angul

(B) Khordha

(C) Sambalpur

(D) Ganjam

[Odisha B.Ed Exam 2018]

Ans- A

Join Us-
ଓଡ଼ିଶାର ସମ� ପୂବର୍ ବଷର୍ର �ଶ� ଉ�ର ପାଇଁ WtsApp/Call- 8596976190
Download “Tech Of World” App

• Satkosia Gorge Wildlife Sanctuary is located in the Angul and


Nayagarh districts of Odisha, India.
• It is a 988.30 km² (381.5 sq mi) protected area that was designated
as a tiger reserve in 2007.
• The sanctuary is home to a variety of wildlife, including tigers,
elephants, leopards, sloth bears, crocodiles, and a number of bird
species.
• The sanctuary is also home to the Satkosia Gorge, which is a 22
km (14 mi) long gorge carved by the Mahanadi River.
• ସାତେକାଶିଆ ବନ��ାଣୀ ଅଭୟାରଣ� ଓଡ଼ିଶାର ଅନୁ ଗଳ
ୁ ଓ ନୟାଗଡ଼ ଜି�ାେର ଅବ�ିତ

• ଏହା ଏକ ୯୮୮.୩୦ ବଗ� କିେଲାମିଟର (୩୮୧.୫ ବଗ� ମାଇଲ) ସଂର�ିତ ଅ�ଳ ଯାହାକୁ
୨୦୦୭ େର ବ�ା� ଅଭୟାରଣ� ଭାବେର ମାନ�ତା ଦିଆଯାଇଥ �ଲା ।
• ଅଭୟାରଣ�େର ବାଘ, ହାତୀ, ଚିତାବାଘ, ଭାଲୁ , କୁ�ୀର ଓ ଅେନକ �ଜାତିର ପ�ୀ
ରହିଛ�ି।
• ଏହି ଅଭୟାରଣ�େର ସାତେକାଶିଆ ଘାଟି ମ� ରହିଛି, ଯାହା କି ମହାନଦୀ �ାରା େଖାଦିତ
୨୨ କିେଲାମିଟର (୧୪ ମାଇଲ) ଲ��ା ଘାଟି ଅେଟ ।

14- Which river has its origin from Gonasika hill?

- େଗାନାସି କା ପାହାଡ଼ରୁ େକଉ ଁ ନଦୀର ଉ��ି େହାଇଛି?

A) Mahanadi

B) Baitarni

C) Devi

D) Brahmani

[OSSSC Junior Clerk 2018]

Ans- B

Join Us-
ଓଡ଼ିଶାର ସମ� ପୂବର୍ ବଷର୍ର �ଶ� ଉ�ର ପାଇଁ WtsApp/Call- 8596976190
Download “Tech Of World” App

The Baitarni River is one of the six major rivers of Odisha. It originates
from the Gonasika hills in the Keonjhar district and flows for about 360
kilometers before draining into the Bay of Bengal. The river is a major
source of water for irrigation and drinking water in the Keonjhar,
Sundargarh, and Jharsuguda districts.

େବ�ତରଣୀ ନଦୀ ଓଡ଼ିଶାର ଛଅଟି �ମୁଖ ନଦୀ ମ�ରୁ ଅନ�ତମ । ଏହା େକ�ୁ ଝର ଜି�ାର
େଗାନାସି କା ପାହାଡ଼ରୁ ଉ�� େହାଇ �ାୟ ୩୬୦ କିେଲାମିଟର ପଯ��� �ବାହିତ େହାଇ
ବେ�ାପସାଗରେର �ବାହିତ େହାଇଥାଏ। େକ�ୁ ଝର, ସୁ�ରଗଡ଼ ଓ ଝାରସୁଗଡ଼
ୁ ା ଜି�ାେର
ଜଳେସଚନ ଓ ପାନୀୟ ଜଳ ପାଇ ଁ ଏହି ନଦୀ ଜଳର ଏକ �ମୁଖ ଉ�।

15- Which of the following options is NOT a tributary of the


Mahanadi river?

- ନି�ଲ
� ି ଖତ
� ବିକ� ଗୁଡ଼କ
ି ମ�ରୁ େକଉଟିଁ ମହାନଦୀର ଶାଖା ନଦୀ ନୁ େହ?ଁ

A) Jonk

B) Betwa

C) Seonath

D) Tel

[Odisha High School Teacher Exam 2021]

Ans- B

Join Us-
ଓଡ଼ିଶାର ସମ� ପୂବର୍ ବଷର୍ର �ଶ� ଉ�ର ପାଇଁ WtsApp/Call- 8596976190
Download “Tech Of World” App

Here are some of the notable tributaries of the Mahanadi River:

1. Seonath River: It is one of the major tributaries of the Mahanadi


and originates in Chhattisgarh. The Seonath River joins the
Mahanadi near Hariharpur in Chhattisgarh.
2. Jonk River: This tributary originates in Chhattisgarh and merges
with the Mahanadi River near Sambalpur in Odisha.
3. Hasdeo River: The Hasdeo River also originates in Chhattisgarh
and joins the Mahanadi River near Sarangarh in Chhattisgarh.
4. Ib River: It originates in the Bastar Plateau of Chhattisgarh and
joins the Mahanadi River near Sonepur in Odisha.
5. Ong River: The Ong River originates in Chhattisgarh and merges
with the Mahanadi River near Cuttack in Odisha.
6. Tel River: Another major tributary, the Tel River, originates in
Chhattisgarh and joins the Mahanadi near Sonepur in Odisha.
7. Hati River: The Hati River is a tributary of the Mahanadi River that
originates in Odisha and joins the main river near Naraj in Odisha.
ଏଠାେର ମହାନଦୀର େକେତକ ଉେ�ଖନୀୟ ଶାଖା ନଦୀ ରହିଛି:
1. େସଓନାଥ ନଦୀ: ଏହା ମହାନଦୀର ଅନ�ତମ �ମୁଖ ଶାଖା ନଦୀ ଏବଂ ଏହା
ଛତିଶଗଡ଼େର ଉ�� େହାଇଛି । ଛତିଶଗଡ଼ର ହରି ହରପୁର ନିକଟେର ମହାନଦୀେର
ି ି େସଓନାଥ ନଦୀ ।
ମିଶଛ
2. ଜ� ନଦୀ: ଏହି ଶାଖା ନଦୀ ଛତିଶଗଡ଼ରୁ ଉ�� େହାଇ ଓଡ଼ିଶାର ସମ�ଲପୁର ନିକଟ�
ମହାନଦୀ ସହିତ ମିଶଯ
ି ାଇଛି।
3. ହାସେଦଓ ନଦୀ: ହାସେଦଓ ନଦୀ ମ� ଛତିଶଗଡ଼ରୁ ଉ�� େହାଇ ଛତିଶଗଡ଼ର
ସାରଣଗଡ଼ ନିକଟେର ମହାନଦୀେର ମିଶଥ
ି ାଏ।
4. ଇବ୍ ନଦୀ: ଛତିଶଗଡ଼ର ବ�ର ମାଳଭୂମିରୁ ଉ�� େହାଇ ଓଡ଼ିଶାର େସାନପୁର ନିକଟ
ମହାନଦୀେର ମିଶଥ
ି ାଏ।
5. ଓ� ନଦୀ: ଓ� ନଦୀ ଛତିଶଗଡ଼ରୁ ଉ�� େହାଇ ଓଡ଼ିଶାର କଟକ ନିକଟ�
ମହାନଦୀେର ମିଶଛି ି।
6. େତଲ ନଦୀ: ଆଉ ଏକ �ମୁଖ ଶାଖା ନଦୀ େତଲ ନଦୀ ଛତିଶଗଡ଼ରୁ ବାହାରି ଓଡ଼ିଶାର
େସାନପୁର ନିକଟେର ମହାନଦୀେର ମିଶଛ ି ି।
7. ହାତୀ ନଦୀ: ହାତୀ ନଦୀ ଓଡ଼ିଶାେର ଉ�� େହାଇ ଓଡ଼ିଶାର ନରାଜ ନିକଟେର ଥ �ବା
ମୁଖ� ନଦୀେର ମିଶଥି �ବା ମହାନଦୀର ଏକ ଶାଖା ନଦୀ ।

To Continue Download The App- Click Here

Join Us-
ଓଡ଼ିଶାର ସମ� ପୂବର୍ ବଷର୍ର �ଶ� ଉ�ର ପାଇଁ
WtsApp/Call- 8596976190
Download “Tech Of World” App

ALL ODISHA COMPUTER PYQ


E- Book (PDF)
All Odisha Exam Previous Year Chapter Wise
COMPUTER Questions & Answer With
EXPLANATION
Language- ODIA & ENGLISH
[Exam- OSSSC, OSSC, OPSC, Odisha Police SI &
Constable, RHT,OSSSC (PEO, RI, AMIN, LSI, OFDC,
Group C)]

Tech Of World APP

Features Of “COMPUTER” Chapter Wise PYQ E-Book

Total PYQ 2,577+ (With Explanation)

Total Topic 15

Language Odia & English

Explanation Odia & English Language

Best For All Odisha Exam- OSSSC, OSSC , OPSC, Police/


Constable, Battalion, JT, RHT, OAVS, & Other Exams
Download App Click Here
WhatsApp / Call 8596976190

Click Here

Join Us-
ଓଡ଼ିଶାର ସମ� ପୂବର୍ ବଷର୍ର �ଶ� ଉ�ର ପାଇଁ
WtsApp/Call- 8596976190
Download “Tech Of World” App

Sl. No. Topic No. Of MCQ

1 Shortcut Key & Full Form 288

2 Internet and its Services 294


3 MS Excel 291
4 MS Word 379
5 MS PowerPoint 137
6 Computer Hardware & Software 226
7 Computer Memory 187
8 Miscellaneous 407
9 Data Communication and Networking 93
10 Operating System 99
11 Programming Concepts 49
12 Windows 33
13 Database Concepts 24
14 Computer Security 15
15 Basic & Introduction 55
Total 2,577

Join Us-
ଓଡ଼ିଶାର ସମ� ପୂବର୍ ବଷର୍ର �ଶ� ଉ�ର ପାଇଁ
WtsApp/Call- 8596976190
Download “Tech Of World” App

Topic Name- “ MS Word ”

1- What is MS Word?

ଏମଏସ ୱାଡର୍ କ'ଣ?

[A] A big word


[B] A dictionary
[C] A word processor
[D] A computer name
[Odisha District Police Constable 2023]

Ans- C
MS Word, short for Microsoft Word, is a popular word processing software
developed and published by Microsoft Corporation. It is a part of the Microsoft
Office suite, which includes other popular software applications like Excel,
PowerPoint, and Outlook.

ମାଇେ�ାସଫ� ୱାଡର୍ ପାଇଁ ସଂ�ି� ଏମଏସ ୱାଡର୍, ଏକ େଲାକ�ିୟ ଶ� ��ିୟାକରଣ ସଫ�େୱୟାର ଯାହା
ମାଇେ�ାସଫ� କେପର୍ାେରସନ �ାରା ବିକଶିତ ଏବଂ �କାଶିତ । ଏହା ମାଇେ�ାସଫ� ଅଫି ସ ସୁଇଟ୍ ର ଏକ
ଅଂଶ, େଯଉଁଥିେର ଅନ୍ୟ େଲାକ�ିୟ ସଫ�େୱୟାର୍ ଆପି�େକସନ୍ େଯପରିକି Excel, PowerPoint
ଏବଂ Outlook ଅ�ଭର୍ୁ�।

Join Us-
ଓଡ଼ିଶାର ସମ� ପୂବର୍ ବଷର୍ର �ଶ� ଉ�ର ପାଇଁ
WtsApp/Call- 8596976190
Download “Tech Of World” App

2- Word processing, Spreadsheet, and photo-editing are examples


of

- ୱାଡର୍ େ�ାେସସିଂ, େ��ଡସିଟ୍ ଏବଂ ଫେଟା-ଏଡିଟଂି ____ର ଉଦାହରଣ

A. application software
B. system software
C. operating system software
D. platform software
[Odisha Jail Warder 2022]

Ans- A
Word processing, spreadsheet, and photo-editing are examples of software
applications
Word processing software like Microsoft Word, Google Docs, or LibreOffice
Writer is used for creating, editing, and formatting text-based documents such
as letters, reports, and resumes.
Spreadsheet software like Microsoft Excel, Google Sheets, or LibreOffice Calc
is used for organizing, analyzing, and manipulating data in the form of rows
and columns.
Photo-editing software like Adobe Photoshop, GIMP, or PaintShop Pro is used
for editing and manipulating digital images, including resizing, cropping,
adjusting color, and adding effects.

Join Us-
ଓଡ଼ିଶାର ସମ� ପୂବର୍ ବଷର୍ର �ଶ� ଉ�ର ପାଇଁ
WtsApp/Call- 8596976190
Download “Tech Of World” App

ୱାଡର୍ େ�ାେସସିଂ, େ�ଡ


� ସିଟ୍ ଏବଂ ଫେଟା-ଏଡିଟଂି େହଉଛି ସଫ�େୱର ଆପି�େକସନ୍ ର ଉଦାହରଣ

ୁ ଡ�, କିମା� ଲିେ�ଅଫି ସ ରାଇଟର ଭଳି ୱାଡର୍ େ�ାେସସିଂ ସଫ�େୱୟାର


ମାଇେ�ାସଫ� ୱାଡର୍, ଗୁଗଲ
ଅ�ର, ରିେପାଟର୍ ଏବଂ ରିଜୟୁ େମ ଭଳି େଟ�ଟ୍ ଆଧାରିତ ଡକୁ େମ� ତିଆରି, ସ�ାଦନା ଏବଂ ଫମର୍ାଟିଂ
ପାଇଁ ବ୍ୟବହୃ ତ ହୁ ଏ ।

ମାଇେ�ାସଫ� ଏେ�ଲ୍, ଗୁଗୁଲ୍ ସିଟ୍ ସ କିମ�ା ଲିେ�ଅଫି ସ କାଲ୍ ସି ଭଳି େ�ଡ୍


� ସିଟ୍ ସଫ�େୱୟାର୍ ଧାଡ଼ି
ଏବଂ �� ଆକାରେର ଡାଟା ସଂଗଠିତ, ବିେ�ଷଣ ଏବଂ ପରିଚାଳନା ପାଇଁ ବ୍ୟବହୃ ତ ହୁ ଏ ।

ଆେଡାବ ଫେଟାସପ୍, ଜିଆଇଏମପି, କିମ�ା େପ�ସପ୍ େ�ା ପରି ଫେଟା-ଏଡିଟଂି ସଫ�େୱୟାର୍ ଡିଜଟ
ି ାଲ
ଛବିଗୁଡ଼କ
ି ର ସ�ାଦନା ଏବଂ ପରିଚାଳନା ପାଇଁ ବ୍ୟବହୃ ତ ହୁ ଏ, େଯଉଁଥିେର ପୁନରୁ�ାର, ଫସଲ, ର�
ଆଡଜ� କରିବା ଏବଂ �ଭାବ େଯାଡିବା ଅ�ଭର୍ୁ� ।

3- What will Shift + F7 do in a Word application?

- ଏକ ୱାଡର୍ ଆପିେ� କସନେର ସିଫ� + F7 କ'ଣ କରିବ?

(A) Open the webpage


(B) Open the hyperlink
(C) Open the thesaurus
(D) Open the image
[OSSC BSSO 2022]

Ans- C

Join Us-
ଓଡ଼ିଶାର ସମ� ପୂବର୍ ବଷର୍ର �ଶ� ଉ�ର ପାଇଁ
WtsApp/Call- 8596976190
Download “Tech Of World” App

In a Word application, pressing Shift + F7 will bring up the Thesaurus pane.


The Thesaurus is a tool in Microsoft Word that allows you to find synonyms
(words with similar meanings) and antonyms (words with opposite meanings)
for the selected word or phrase.

ଏକ ୱାଡର୍ ଆପି�େକସନେର, ଶିଫ� + F7 ଦବାଇେଲ ଥିେସାରସ୍ େପନ୍ ଆସିବ । ଥିେସାରସ େହଉଛି


ମାଇେ�ାସଫ� ୱାଡର୍ର ଏକ ଉପକରଣ ଯାହା ଆପଣ�ୁ ଚୟନିତ ଶ� କିମା� ବାକ୍ୟ ପାଇଁ ସମାଥର୍କ ଶ�
(ସମାନ ଅଥର୍ ଥିବା ଶ�) ଏବଂ ବିେଲାମ (ବିପରୀତ ଅଥର୍ ଥିବା ଶ�) େଖାଜିବାକୁ ଅନୁ ମତି ଦିଏ ।

4- To create a newspaper-like layout in your Word document, you


need to choose _______ option from the Layout tab.

ଆପଣ� ୱାଡର୍ ଡକୁ େମ� (Word document) େର ଖବରକାଗଜ ଭଳି େଲଆଉଟ (layout)
��ୁ ତ କରିବା ପାଇଁ ଆପଣ େଲଆଉଟ ଟ୍ୟାବ (Layout tab) ରୁ _______ ଅପସନ ଚୟନ
କରିବା ଆବଶ୍ୟକ ।

(A) Border
(B) Orientation
(C) Column
(D) Size
[OSSC RHT High School Teacher 2023]

Ans- C
The Columns option allows you to divide your document into multiple columns,
just like a newspaper. You can choose the number of columns you want, the width
of each column, and the spacing between columns.

�� ବିକ� ଆପଣ�ୁ ଏକ ଖବରକାଗଜ ପରି ଆପଣ� ଡକୁ େମ�କୁ ଏକାଧିକ ��େର ବିଭ� କରିବାକୁ
ଅନୁ ମତି ଦିଏ। ଆପଣ ଚାହଁୁଥିବା �� ସଂଖ୍ୟା, �େତ୍ୟକ ��ର �� ଏବଂ �� ମଧ୍ୟେର ବ୍ୟବଧାନ ଚୟନ
କରିପାରିେବ।

Join Us-
ଓଡ଼ିଶାର ସମ� ପୂବର୍ ବଷର୍ର �ଶ� ଉ�ର ପାଇଁ
WtsApp/Call- 8596976190
Download “Tech Of World” App

5- How to go to Page number 345 in a MS-WORD document


containing 465 pages?

- 465 ପୃ�ା ବିଶ�


ି ଏମଏସ-ୱାଡର୍ ଡକୁ ୍ୟେମ�େର ପୃ�ା ନମ�ର 345କୁ କିପରି ଯିେବ?

A) Press Ctrl + F, then enter 345 and press enter


B) Press Ctrl+ R, then enter 345 and press enter
C) Press Alt+G, then enter 345 and press enter
D) Press Ctrl+G, then enter 345 and press enter
[OSSSC Junior Clerk 2018]

Ans- D

• Press the "Ctrl" + "G" keys on your keyboard to open the "Go To" dialog
box.
• In the "Go To" dialog box, type in "345" in the "Page" field and click on the
"Go To" button.
• MS Word will jump to page 345 in your document.

• "େଗା ଟୁ " ଡାଏଲଗ ବ� େଖାଲିବାକୁ ଆପଣ� କୀେବାଡର୍େର "Ctrl" + "G" କୀ ଦବା�ୁ ।


• "େଗା ଟୁ " ଡାଏଲଗ ବ�େର, "ପୃ�ା" େ��େର "345" ଟାଇପ୍ କର�ୁ ଏବଂ "େଗା ଟୁ " ବଟନେର କି�କ୍
କର�ୁ ।
• MS ଶ� ଆପଣ� ଡକୁ େମ�ର ପୃ�ା 345କୁ େଡଇଁବ।

Join Us-
ଓଡ଼ିଶାର ସମ� ପୂବର୍ ବଷର୍ର �ଶ� ଉ�ର ପାଇଁ
WtsApp/Call- 8596976190
Download “Tech Of World” App

6- _________ option enables you to add common text at the top of


all the pages in a Word document.

- _____ ବିକ� ଆପଣ�ୁ ଏକ ୱାଡର୍ ଡକୁ ୍ୟେମ�େର ସମ� ପୃ�ାର ଶୀଷର୍େର ସାଧାରଣ େଟ�ଟ୍
େଯାଡିବାକୁ ସ�ମ କେର।

(A) Footer
(B) Header
(C) Dropcap
(D) Wordwrap
[OSSC CGL Exam 2022]

Ans- B

A header is a section of a document that appears at the top of each page and
contains information such as page numbers, document title, author name,
company logo, and other relevant details.

ଶୀଷର୍କ େହଉଛି ଏକ ଡକୁ େମ�ର ଏକ ବିଭାଗ ଯାହା �େତ୍ୟକ ପୃ�ାର ଶୀଷର୍େର େଦଖାଯାଏ ଏବଂ
ଏଥିେର ପୃ�ା ସଂଖ୍ୟା, ଡକୁ େମ� ଶୀଷର୍କ, େଲଖକ ନାମ, କ�ାନୀ େଲାେଗା ଏବଂ ଅନ୍ୟାନ୍ୟ �ାସ�ି କ
ବିବରଣୀ ପରି ସୂଚନା ରହିଥାଏ ।

Join Us-
ଓଡ଼ିଶାର ସମ� ପୂବର୍ ବଷର୍ର �ଶ� ଉ�ର ପାଇଁ
WtsApp/Call- 8596976190
Download “Tech Of World” App

7- Which key should be pressed to start a new paragraph in MS-


Word?

- ଏମଏସ-ୱାଡର୍େର ଏକ ନୂ ତନ ପାରା�ାଫ୍ ଆର� କରିବା ପାଇଁ େକଉଁ key ଦବାଇବା ଉଚିତ୍?

A) Down Cursor Key


B) Enter Key
C) Shift + Enter
D) Ctrl + Enter
[OSSSC Junior Clerk 2018]

Ans- B

Pressing the "Enter" key creates a new line and moves the cursor to the
beginning of the next line, effectively starting a new paragraph. This is the
standard way to create paragraphs in most word processing software, including
MS-Word.

"ଏ�ର" କୀ ଦବାଇବା �ାରା ଏକ ନୂ ତନ େରଖା ସୃ�ି ହୁ ଏ ଏବଂ କସର୍ରକୁ ପରବ�ର୍ୀ ଧାଡ଼ିର �ାର�କୁ
�ାନା�ରିତ କରାଯାଏ, �ଭାବଶାଳୀ ଭାବେର ଏକ ନୂ ତନ ପାରା�ାଫ୍ ଆର� ହୁ ଏ । ଏମଏସ-ୱାଡର୍ ସେମତ
ଅଧିକାଂଶ ୱାଡର୍ େ�ାେସସିଂ ସଫ�େୱୟାରେର ପାରା�ାଫ୍ ତିଆରି କରିବାର ଏହା ଏକ ମାନକ ଉପାୟ ଅେଟ

Join Us-
ଓଡ଼ିଶାର ସମ� ପୂବର୍ ବଷର୍ର �ଶ� ଉ�ର ପାଇଁ
WtsApp/Call- 8596976190
Download “Tech Of World” App

8- Which one of these does NOT belong to MS Office?

- ଏମାନ� ମଧ୍ୟରୁ େକଉଁଟ ି ଏମଏସ ଅଫି ସର ନୁ େହଁ?

A) PowerPoint
B) Excel
C) Word
D) Notepad
[OSSC Junior Assistant 2022 Exam]

Ans- D

Notepad is a basic text editor program that comes pre-installed on Windows


operating systems. It is a simple tool for creating and editing plain text files, but it
does not have the advanced formatting and layout options of the other programs
in MS Office, such as Word, Excel, and PowerPoint.

େନାଟ�୍ୟାଡ୍ େହଉଛି ଏକ େମୗଳିକ େଟ�ଟ୍ ଏଡିଟର େ�ା�ାମ ଯାହା ୱିେ�ାଜ୍ ଅପେରଟିଂ ସି�ମ୍ େର �ି-
ଇନ�ଲ୍ େହାଇଥାଏ। ଏହା ସାଦା େଟ�ଟ୍ ଫାଇଲ୍ ସୃ�ି ଏବଂ ସ�ାଦନା ପାଇଁ ଏକ ସରଳ ଉପକରଣ, କି�ୁ
ି ର ଉ�ତ ଫମର୍ାଟିଂ ଏବଂ େଲଆଉଟ୍ ବିକ� ନାହିଁ,
ଏଥିେର MS ଅଫି ସ୍ େର ଥିବା ଅନ୍ୟ େ�ା�ାମଗୁଡକ
େଯପରିକି ୱାଡର୍, ଏେ�ଲ ଏବଂ ପାୱାରପଏ�।

Join Us-
ଓଡ଼ିଶାର ସମ� ପୂବର୍ ବଷର୍ର �ଶ� ଉ�ର ପାଇଁ
WtsApp/Call- 8596976190
Download “Tech Of World” App

9- Which among the following is not a Video Conferencing Tool ?

- ନିମ�ଲିଖିତ ମଧ୍ୟରୁ େକଉଁଟ ି ଭିଡଓ


ି କନଫେରନି�ଂ ଟୁ ଲ୍ ନୁ େହଁ ?

A) Zoom Meetings
B) MS Teams
C) Google Meet
D) MS Outlook
[Odisha High School Teacher Exam 2022]

Ans- D

MS Outlook is an email and calendar management software developed by


Microsoft. It is used for sending and receiving emails, managing contacts and
calendars, and scheduling meetings.

MS Outlook େହଉଛି ମାଇେ�ାସଫ� �ାରା ବିକଶିତ ଏକ ଇେମଲ୍ ଏବଂ କ୍ୟାେଲ�ର ପରିଚାଳନା


ସଫ�େୱୟାର୍ । ଏହା ଇେମଲ୍ ପଠାଇବା ଏବଂ �ା� କରିବା, ସ�କର୍ ଏବଂ କ୍ୟାେଲ�ର ପରିଚାଳନା କରିବା
ଏବଂ େବୖଠକର ସମୟ ନି�ାର୍ ରଣ ପାଇଁ ବ୍ୟବହୃ ତ ହୁ ଏ।

Join Us-
ଓଡ଼ିଶାର ସମ� ପୂବର୍ ବଷର୍ର �ଶ� ଉ�ର ପାଇଁ
WtsApp/Call- 8596976190
Download “Tech Of World” App

10- The quickest and easiest way in Word, to locate a particular


word or phrase in a document is to use the command:

- ଏକ ଡକୁ େମ�େର ଏକ ନି��


ିର୍ ଶ� କିମ�ା ବାକ୍ୟ ଚି�ଟ କରିବା ପାଇଁ ଶ�ର ସବୁ ଠାରୁ ଦ� ତ ଏବଂ ସହଜ
ଉପାୟ େହଉଛି କମା� ବ୍ୟବହାର କରିବା:

(A) Replace
(B) Find
(C) Lookup
(D) Search
[OSSSC RI Exam 2021]

Ans- B
To use the "Find" command in Word, you can follow these steps:
• Open the Word document that you want to search.
• Press "Ctrl" + "F" on your keyboard to open the "Find and Replace" dialog box.
• In the "Find what" field, type the word or phrase that you want to search for.
• Click on the "Find Next" button to search for the first occurrence of the word or
phrase.

ୱାଡର୍େର "ସ�ାନ" କମା� ବ୍ୟବହାର କରିବାକୁ , ଆପଣ ଏହି ପଦେ�ପଗୁଡକ


ି ଅନୁ ସରଣ କରିପାରିେବ:

• ଆପଣ ସ�ାନ କରିବାକୁ ଚାହଁୁଥିବା ୱାଡର୍ ଡକୁ ୍ୟେମ� େଖାଲ�ୁ ।

• "ସ�ାନ ଏବଂ ବଦଳା�ୁ " ଡାଏଲଗ ବ� େଖାଲିବାକୁ ଆପଣ� କୀେବାଡର୍େର "Ctrl" + "F" ଦବା�ୁ ।

• "କ'ଣ ସ�ାନ କର�ୁ " େ��େର, ଆପଣ ସ�ାନ କରିବାକୁ ଚାହଁୁଥିବା ଶ� କିମା� ବାକ୍ୟ ଟାଇପ୍ କର�ୁ ।

• ଶ� କିମ�ା ବାକ୍ୟର �ଥମ ଘଟଣା ସ�ାନ କରିବାକୁ "ପରବ�ର୍ୀ ସ�ାନ" ବଟନେର କି�କ୍ କର�ୁ ।

Join Us-
ଓଡ଼ିଶାର ସମ� ପୂବର୍ ବଷର୍ର �ଶ� ଉ�ର ପାଇଁ
WtsApp/Call- 8596976190
Download “Tech Of World” App

11- Which among the following print commands should be selected


to print first 5 pages of document:

- ଡକୁ େମ�ର �ଥମ 5 ପୃ�ା ଛାପିବା ପାଇଁ ନିମଲ


� ିଖିତ �ି� କମା�ମଧ୍ୟରୁ େକଉଁଟ ି ଚୟନ କରାଯିବା
ଉଚିତ୍:

(A) Print All


(B) From_to_
(C) Page Setup
(D) Page Control
[OSSSC Combined Recruitment 2022]

Ans- B

The "Print All" command will print all pages of the document, while the "Page
Setup" command is used to adjust the page settings such as margins, orientation,
and size. The "Page Control" command is not a standard print command in
Word.

"�ି� ଅଲ୍" କମା� ଡକୁ େମ�ର ସମ� ପୃ�ାଗୁଡକ


ି ମୁ�ଣ କରିବ, େଯେତେବେଳ କି "ପୃ�ା େସଟଅପ୍"
କମା� ମାଜିର୍ନ୍, ଓରିଏେ�ସନ୍ ଏବଂ ଆକାର ପରି ପୃ�ା େସଟିଂସ୍ କୁ ଆଡଜ� କରିବାକୁ ବ୍ୟବହୃ ତ ହୁ ଏ।
"ପୃ�ା ନିୟ�ଣ" କମା� ୱାଡର୍େର ଏକ �ା�ାଡର୍ �ି� କମା� ନୁ େହଁ।

Join Us-
ଓଡ଼ିଶାର ସମ� ପୂବର୍ ବଷର୍ର �ଶ� ଉ�ର ପାଇଁ
WtsApp/Call- 8596976190
Download “Tech Of World” App

12- In print preview, which one of the below actions cannot be


performed?

- �ି� �ିଭୁ୍ୟେର, ନିମ�ଲିଖିତ �ିୟାଗୁଡକ


ି ମଧ୍ୟରୁ େକଉଁଟ ି ସ�ାଦିତ େହାଇପାରିବ ନାହିଁ?

A) Changing Margins
B) Change orientation of page
C) Zoom in and zoom out
D) Edit the document
[OSSC Junior Clerk 2022]

Ans- D

In print preview, you can perform the following actions:


A) Changing margins: You can adjust the margins of the document to
change the layout of the content on the page.
B) Change orientation of page: You can switch the orientation of the
page between portrait and landscape mode to change the page layout.
C) Zoom in and zoom out: You can zoom in or out to see a larger or
smaller preview of the page.
�ି� �ିଭୁ୍ୟେର, ଆପଣ ନିମଲ
� ିଖିତ �ିୟାଗୁଡକ
ି ସ�ାଦନ କରିପାରିେବ:
A) ମାଜିର୍ନ୍ ପରିବ�ର୍ନ କରିବା: ଆପଣ ପୃ�ାେର ବିଷୟବ�ୁ ର େଲଆଉଟ୍ ପରିବ�ର୍ନ କରିବାକୁ
ଡକୁ େମ�ର ମାଜିର୍ନ୍ ଆଡଜ� କରିପାରିେବ।
B) ପୃ�ାର ଓରିଏେ�ସନ୍ ପରିବ�ର୍ନ କର�ୁ : ଆପଣ ପୃ�ା େଲଆଉଟ୍ ପରିବ�ର୍ନ କରିବାକୁ େପାେ�ଟ୍
ଏବଂ ଲ୍ୟା�େ�ପ େମାଡ୍ ମଧ୍ୟେର ପୃ�ାର ଓରିଏେ�ସନ୍ ପରିବ�ର୍ନ କରିପାରିେବ।
C) ଜୁ ମ୍ ଇନ୍ ଏବଂ ଜୁ ମ୍ ଆଉଟ୍: ଆପଣ ପୃ�ାର ଏକ ବଡ଼ କିମା� େଛାଟ �ିଭୁ୍ୟ େଦଖିବା ପାଇଁ ଜୁ ମ୍ ଇନ୍
କିମା� ବାହାରକୁ ଯାଇପାରିେବ।

Join Us-
ଓଡ଼ିଶାର ସମ� ପୂବର୍ ବଷର୍ର �ଶ� ଉ�ର ପାଇଁ
WtsApp/Call- 8596976190
Download “Tech Of World” App

13- How many ways you can save a document?


- ଆପଣ ଏକ ଡକୁ େମ�କୁ େକେତ ଉପାୟେର ସ�ୟ କରିପାରିେବ?

A- 3
B- 4
C- 2
D- 1
[Odisha Police SI 2022 Exam]

Ans- A

• By clicking File on the top left corner and then click save as after that
browse the location where exactly you want to save on your computer.
• By just pressing Ctrl + S and then browse the location where you want to
save.
• By pressing F12 and then browse the location where you want to save.

• ଉପର ବାମ େକାଣେର ଫାଇଲ୍ କିକ୍


� କରି ଏବଂ ତା'ପେର େସଭ୍ କିକ୍
� କର�ୁ ଏବଂ ଏହା ପେର େସଭ୍ କିକ୍

କର�ୁ ଯାହା ପେର ଆପଣ ଆପଣ� କ�ୁ୍ୟଟରେର �କୃ ତେର ସ�ୟ କରିବାକୁ ଚାହଁୁଥିବା �ାନ�ାଉଜ୍
କର�ୁ ।
• େକବଳ Ctrl + S ଦବା �ାରା ଏବଂ ତା'ପେର ଆପଣ ସ�ୟ କରିବାକୁ ଚାହଁୁଥିବା �ାନ�ାଉଜ୍ କର�ୁ ।
• F12 ଦବାଇ ଏବଂ ତା'ପେର ଆପଣ ସ�ୟ କରିବାକୁ ଚାହଁୁଥିବା �ାନ�ାଉଜ୍ କର�ୁ ।

14- In MS Word, which of the following do you use to change


margins?

- ଏମଏସ ୱାଡର୍େର, ଆପଣ ମାଜିର୍ନ ପରିବ�ର୍ନ କରିବାକୁ ନିମଲ


� ିଖିତ ମଧ୍ୟରୁ େକଉଁଟ ି ବ୍ୟବହାର କର�ି?

A) formatting toolbar
B) page setup dialog box
C) standard toolbar

Join Us-
ଓଡ଼ିଶାର ସମ� ପୂବର୍ ବଷର୍ର �ଶ� ଉ�ର ପାଇଁ
WtsApp/Call- 8596976190
Download “Tech Of World” App

D) paragraph dialog box


[OSSSC Junior Clerk 2018]

Ans- B

In MS Word, you use the Page Setup dialog box to change margins. This dialog
box provides options for setting margins, orientation, paper size, and other
page layout settings. To access the Page Setup dialog box, you can either click
on the Page Setup launcher in the Page Setup group on the Page Layout tab, or
go to the File menu, select Page Setup, and then select the Margins tab.

MS ୱାଡର୍େର, ଆପଣ ମାଜିର୍ନ୍ ପରିବ�ର୍ନ କରିବାକୁ ପୃ�ା େସଟଅପ୍ ଡାଏଲଗ୍ ବ� ବ୍ୟବହାର କର�ି।
ଏହି ଡାଏଲଗ ବ� ମାଜିର୍ନ୍, ଓରିଏେ�ସନ୍, କାଗଜ ଆକାର ଏବଂ ଅନ୍ୟ ପୃ�ା େଲଆଉଟ୍ େସଟିଂସ୍ େସଟ୍
କରିବା ପାଇଁ ବିକ� �ଦାନ କେର। ପୃ�ା େସଟଅପ୍ ଡାଏଲଗ୍ ବ�ଆକେସସ୍ କରିବାକୁ , ଆପଣ ପୃ�ା
େଲଆଉଟ୍ ଟ୍ୟାବ୍ େର ପୃ�ା େସଟଅପ୍ ଗ�ପ୍ େର ଥିବା ପୃ�ା େସଟଅପ୍ ଲ�ର୍ ଉପେର କିକ୍
� କରିପାରିେବ
, କିମା� ଫାଇଲ୍ େମନୁ ୍ୟକୁ ଯାଇ, ପୃ�ା େସଟଅପ୍ ଚୟନ କରିପାରିେବ, ଏବଂ ତା'ପେର ମାଜିର୍ନ୍ ଟ୍ୟାବ୍
ଚୟନ କରିପାରିେବ।

15- What programme is used in MS Word to check the spellings?

- ବନାନ ଯା� କରିବା ପାଇଁ ଏମଏସ ୱାଡର୍େର େକଉଁ େ�ା�ାମ ବ୍ୟବହାର କରାଯାଏ?

(A) Outlook Express


(B) Spell-check
(C) Speller
(D) None of above
[OSSSC Combined Recruitment 2022]

Join Us-
ଓଡ଼ିଶାର ସମ� ପୂବର୍ ବଷର୍ର �ଶ� ଉ�ର ପାଇଁ
WtsApp/Call- 8596976190
Download “Tech Of World” App

Ans- B

• In MS Word, the program used to check spelling is called "Spelling and Grammar
Checker".
• To use the Spelling and Grammar Checker, you can either click on the "Spelling
and Grammar" button in the Proofing group on the Review tab, or press "F7" on
your keyboard.

• ଏମଏସ ୱାଡର୍େର, ବନାନ ଯା� କରିବା ପାଇଁ ବ୍ୟବହୃ ତ େ�ା�ାମକୁ "େ�ଲି ଂ ଏବଂ ବ୍ୟାକରଣ େଚକର"
କୁ ହାଯାଏ ।

• େ�ଲିଂ ଏବଂ ବ୍ୟାକରଣ େଚକର ବ୍ୟବହାର କରିବାକୁ , ଆପଣ ସମୀ�ା ଟ୍ୟାବ୍ େର ଥିବା ପ�ଫିଂ ଗ�ପ୍ େର
ଥିବା "େ�ଲି ଂ ଏବଂ ବ୍ୟାକରଣ" ବଟନେର କି�କ୍ କରିପାରିେବ, କିମା� ଆପଣ� କିେବାଡର୍େର "F7" ଦବା�ୁ ।

16- In MS Word, which shortcut key is used to print a document?

- ଏମଏସ ୱାଡର୍େର, ଏକ ଡକୁ େମ� ଛାପିବା ପାଇଁ େକଉଁ ସଟର୍କଟ୍ key ବ୍ୟବହାର କରାଯାଏ?

(A) Shift + P
(B) Alt + P
(C) Tab + P
(D) Ctrl + P
[Odisha Police ASI 2022]

Ans- D

Join Us-
ଓଡ଼ିଶାର ସମ� ପୂବର୍ ବଷର୍ର �ଶ� ଉ�ର ପାଇଁ
WtsApp/Call- 8596976190
Download “Tech Of World” App

• In MS Word, the shortcut key used to print a document is "Ctrl + P". This
keyboard shortcut opens the Print dialog box, where you can choose the printer,
set print options such as number of copies, page range, and paper size, and then
print the document.
• This shortcut is often quicker than going to the File menu and selecting Print, or
using the Print button on the Quick Access Toolbar.

• ଏମଏସ ୱାଡର୍େର, ଏକ ଡକୁ େମ� ଛାପିବା ପାଇଁ ବ୍ୟବହୃ ତ ସଟର୍କଟ କୀ େହଉଛି "Ctrl + P"। ଏହି
କୀେବାଡର୍ ସଟର୍କଟ୍ �ି� ଡାଇଲଗ୍ ବ� େଖାଲିଥାଏ, େଯଉଁଠାେର ଆପଣ �ି�ର୍ ଚୟନ କରିପାରିେବ, କପି
ସଂଖ୍ୟା, ପୃ�ା ସୀମା ଏବଂ କାଗଜ ଆକାର ଭଳି �ି� ବିକ� େସଟ୍ କରିପାରିେବ ଏବଂ ତା'ପେର ଡକୁ େମ�କୁ
�ି� କରିପାରିେବ।

• ଫାଇଲ୍ େମନୁ ୍ୟକୁ ଯାଇ �ି� ଚୟନ କରିବା କିମ�ା �ିକ୍ ଆେ�ସ୍ ଟୁ ଲବାରେର �ି� ବଟନ୍ ବ୍ୟବହାର କରିବା
ଅେପ�ା ଏହି ସଟର୍କଟ୍ ଅଧିକାଂଶ ସମୟେର ଶୀ� େହାଇଥାଏ।

To Continue- Click Here

Join Us-
ଓଡ଼ିଶାର ସମ� ପୂବର୍ ବଷର୍ର �ଶ� ଉ�ର ପାଇଁ
WtsApp/Call- 8596976190
Download “Tech Of World” App

ALL ODISHA ENGLISH PYQ


E- Book (PDF)
All Odisha Exam Previous Year Chapter Wise
ENGLISH Question
EXPLANATION
[Exam- OSSSC, OSSC, OPSC, Police SI / Constable, Battalion,
Fireman, Jail Warder, B.ED, RHT, CT, JT, OAVS, OTET, OSSTET,
OSSSC (PEO, RI, ARI, JA, LSI, OFDC, Group C) & Other Exams]

Tech Of World APP

Features Of “ENGLISH” Chapter Wise PYQ E-Book


Total PYQ 5,532+

Total Topic 27

Explanation Odia & English Language

Best For All Odisha Exam- OSSSC, OSSC, OPSC, Police SI / Constable,
Battalion, Fireman, Jail Warder, B.ED, RHT, CT, JT, OAVS,
OTET, OSSTET, OSSSC (PEO, RI, ARI, JA, LSI, OFDC, Group C)
& Other Exams
Download App To Buy Click Here

WhatsApp / Call 8596976190

Click Here

Join Us-
ଓଡ଼ିଶାର ସମ� ପୂବର୍ ବଷର୍ର �ଶ� ଉ�ର ପାଇଁ
WtsApp/Call- 8596976190
Download “Tech Of World” App

Sl. No. Topic No. Of MCQ

1 Passage (B.Ed, CHT, CT & Other 810


Teaching Exam )
2 Passage (OSSSC, OSSC, OPSC & Other 571
Exam)
3 Verb 360
4 Preposition 362
5 Synonyms 301
6 Fill In The Blank 252
7 Tense 391
8 Spelling 232
9 Speech Or Narration 168
10 Voice (Active & Passive) 170
11 Idioms & Phrases 182
12 Noun & Pronoun 131
13 Antonyms 286
14 Articles 225
15 Finding Error 239
16 Jumbled Sentence 42
17 One Word Substitution 112
18 Miscellaneous 191
19 Phrasal Verbs 53
20 Phrase 67
21 Suffix & Prefix 53
22 Type Of Sentence 70
23 Pronunciation 16
24 Punctuation 83
25 Adjective & Adverb 80
26 Conjunction 27
27 English Pedagogy 58
Total 5,532

Join Us-
ଓଡ଼ିଶାର ସମ� ପୂବର୍ ବଷର୍ର �ଶ� ଉ�ର ପାଇଁ
WtsApp/Call- 8596976190
Download “Tech Of World” App

Topic Name- Preposition

1- Fill in the blanks with suitable Preposition from the given


alternatives.
[OSSC Traffic SI 2022]
[OSSC Excise SI 2021]
The book is _____________ the table.
(A) across
(B) about
(C) through
(D) on

ANS- D

Explanation: The correct preposition in this sentence is "on." We use


"on" to indicate a position where something is in contact with a surface.
Therefore, the book is on the table.

Additional examples:

1. The keys are on the shelf.


2. The laptop is on the desk.
3. The cat is sitting on the chair.

Join Us-
ଓଡ଼ିଶାର ସମ� ପୂବର୍ ବଷର୍ର �ଶ� ଉ�ର ପାଇଁ
WtsApp/Call- 8596976190
Download “Tech Of World” App

2- Fill in the blanks with suitable Preposition from the given


alternatives.
The squad found a bomb ___________ the truck.
[OSSC SCEW 2022]
[Odisha Police ASI 2022]
(A) underneath
(B) against
(C) for
(D) to

ANS- A

The appropriate preposition to fill in the blank is (A) underneath. The


sentence would be: "The squad found a bomb underneath the truck." The
preposition "underneath" is used to indicate position or location below or
beneath something.

Example:

• She discovered a hidden treasure chest underneath the old oak tree.
• The cat hid underneath the bed during the thunderstorm.

• The keys were found underneath a pile of papers on the desk.

Join Us-
ଓଡ଼ିଶାର ସମ� ପୂବର୍ ବଷର୍ର �ଶ� ଉ�ର ପାଇଁ
WtsApp/Call- 8596976190
Download “Tech Of World” App

3- The woman was suffering _________ cancer, but the doctor


cured her __________ it.
[OSSSC RI Exam 2021]
A) from, of
(B) with, from
(C) from, with
(D) of, from

ANS- A

Explanation: "From" is used to indicate the source or cause of an


ailment or condition, so "from" is used to show that the woman was
suffering from cancer. "Of" is used to show possession or association,
so "of" is used to indicate that the doctor cured her of the cancer.

Examples:

1. He is suffering from a severe headache.


2. She was cured of her fear of heights.
3. The patient is recovering from a case of pneumonia.

Join Us-
ଓଡ଼ିଶାର ସମ� ପୂବର୍ ବଷର୍ର �ଶ� ଉ�ର ପାଇଁ
WtsApp/Call- 8596976190
Download “Tech Of World” App

4- He could not comply _________ my demand.


[OSSSC Combined Recruitment 2022]
(A) upon
(B) against
(C) to
(D) with

ANS- D

Explanation: "With" is used to indicate compliance or agreement. In this


sentence, it is used to show that he could not comply with the speaker's
demand.

Examples:

1. She complied with the teacher's instructions.


2. They refused to comply with the regulations.
3. He complied with his boss's request.

Join Us-
ଓଡ଼ିଶାର ସମ� ପୂବର୍ ବଷର୍ର �ଶ� ଉ�ର ପାଇଁ
WtsApp/Call- 8596976190
Download “Tech Of World” App

5- Fill in the blank using suitable words/phrase given below?


"Trees grow ____________ the bank of the river".
[Odisha District Police Constable 2013]
(A) behind
(B) along
(C) at
(D) across

Ans- B

Explanation: The correct preposition in this sentence is "along." The


phrase "along the bank of the river" indicates the direction or position of
the tree's growth in relation to the river.
Examples:
1. Flowers bloom along the pathway in the garden.
2. The houses are built along the coastline, facing the ocean.
3. The fence runs along the boundary of the property.

Join Us-
ଓଡ଼ିଶାର ସମ� ପୂବର୍ ବଷର୍ର �ଶ� ଉ�ର ପାଇଁ
WtsApp/Call- 8596976190
Download “Tech Of World” App

6- I like to indulge ____________ a hot water bath.


[Odisha Battalion 2017]
A) on
B) in
C) for
D) of

Ans- B
Explanation: The correct preposition in this sentence is "in." The phrase
"indulge in" is used to express the enjoyment or indulgence in a particular
activity.
Examples:
1. He likes to indulge in a good book on weekends.
2. They indulge in their favourite hobbies during their free time.
3. She indulges in delicious desserts occasionally.

Join Us-
ଓଡ଼ିଶାର ସମ� ପୂବର୍ ବଷର୍ର �ଶ� ଉ�ର ପାଇଁ
WtsApp/Call- 8596976190
Download “Tech Of World” App

7- Take this medicine and you will get rid -------- the bad cold
[Odisha Police SI 2022 Exam]
A- of
B- over
C- at
D- from

ANS- A

Explanation: "Of" is used to indicate removal or separation. In this


sentence, it indicates getting rid of the bad cold. Therefore, the correct
preposition is "of."

Examples:

1. She finally got rid of her old clothes.


2. We need to get rid of the clutter in the house.
3. He couldn't get rid of the bad smell in his car.

Join Us-
ଓଡ଼ିଶାର ସମ� ପୂବର୍ ବଷର୍ର �ଶ� ଉ�ର ପାଇଁ
WtsApp/Call- 8596976190
Download “Tech Of World” App

8- _______ veena, she also knows how to play violin and flutes.
[OSSC CGL Exam 2022]
(A) However
(B) Besides
(C) Otherwise
(D) Unless

ANS- B

Explanation: "Besides" is used to indicate an additional fact or element.


In this sentence, it is used to show that besides playing the veena, she
also knows how to play the violin and flutes.

Examples:

1. Besides being a doctor, he is also an excellent painter.


2. Besides studying, she enjoys playing sports.
3. Besides English, he speaks French fluently.

Join Us-
ଓଡ଼ିଶାର ସମ� ପୂବର୍ ବଷର୍ର �ଶ� ଉ�ର ପାଇଁ
WtsApp/Call- 8596976190
Download “Tech Of World” App

9- Fill in the blanks with suitable Preposition from the given


alternatives.
We have been living in this palace ___________ 1956.
[OSSC Investigator 2023]
(A) through
(B) against
(C) since
(D) for

Ans- C

Explanation: "Since" is used to indicate a starting point or time. In this


sentence, it is used to show that the speaker has been living in the
palace from 1956.

Examples:

1. I have known him since childhood.


2. She has been working here since last year.
3. They have been friends since college.

Join Us-
ଓଡ଼ିଶାର ସମ� ପୂବର୍ ବଷର୍ର �ଶ� ଉ�ର ପାଇଁ
WtsApp/Call- 8596976190
Download “Tech Of World” App

10- Fill in the blanks with suitable Preposition from the given
alternatives.
Be nice _________ your parents.
[OSSC JEA 2022]
(A) in
(B) to
(C) at
(D) under

Ans- B

Explanation: The correct preposition in this sentence is "to." The


phrase "nice to" is used to indicate the recipient of the niceness or
kindness.
Examples:
1. He is always kind to his younger siblings.
2. She is respectful and polite to her elders.
3. They are considerate and supportive to their friends.

Join Us-
ଓଡ଼ିଶାର ସମ� ପୂବର୍ ବଷର୍ର �ଶ� ଉ�ର ପାଇଁ
WtsApp/Call- 8596976190
Download “Tech Of World” App

11- They agreed ___________ his proposal in the meeting, but I did
not agree __________ them.
[OSSSC RI Exam 2021]
(A) to, with
(B) with, to
(C) into, with
(D) upon, to

ANS- A

The appropriate prepositions to fill in the blanks are (A) to, with. The
sentence would be: "They agreed to his proposal in the meeting, but I
did not agree with them." The preposition "to" is used to indicate consent
or acceptance, and the preposition "with" is used to indicate agreement
or alignment.

Example:

• We agreed to their terms and signed the contract.


• She disagreed with him on the issue and presented her own
argument.

• They all reached an agreement on the project plan.

Join Us-
ଓଡ଼ିଶାର ସମ� ପୂବର୍ ବଷର୍ର �ଶ� ଉ�ର ପାଇଁ
WtsApp/Call- 8596976190
Download “Tech Of World” App

12- He is indebted ..........his friend ………………. a large sum.


(Choose appropriate prepositions)
[OSSSC Junior Clerk 2018]
A) to, for
B) for, to
C) by, for
D) to to

ANS- A

The appropriate prepositions to fill in the blanks are (A) to, for. The
sentence would be: "He is indebted to his friend for a large sum." The
preposition "to" is used to indicate obligation or indebtedness, and the
preposition "for" is used to indicate the reason or cause of indebtedness.

Example:

• I am deeply indebted to my parents for their support and guidance.


• She is indebted to her mentor for helping her achieve her goals.

• They feel indebted to the community for their continuous support.

Join Us-
ଓଡ଼ିଶାର ସମ� ପୂବର୍ ବଷର୍ର �ଶ� ଉ�ର ପାଇଁ
WtsApp/Call- 8596976190
Download “Tech Of World” App

13- They organized a party ___________ his honour. That Ajita was
not invited ___________ it, was a shame. She admired him most
and it seems unlikely that she would forgive the organizers
_________ their omission.
[OPSC ASO Exam 2022]
(A) at, for, towards
(B) about, at, against
(C) in, to, for
(D) after, on, with

Ans- C

Explanation: The correct prepositions in this sentence are "in," "to," and
"for."
- "In his honour" indicates the purpose or reason for organizing the party.
- "To it" refers to the party mentioned earlier in the sentence.
- "For their omission" indicates the cause or reason for which Ajita might
not forgive the organizers.
Examples:
1. They organized a surprise celebration in her honour on her birthday.
2. The company arranged a farewell party for the retiring employee.
3. The community planned a grand event in the mayor's honour.

Join Us-
ଓଡ଼ିଶାର ସମ� ପୂବର୍ ବଷର୍ର �ଶ� ଉ�ର ପାଇଁ
WtsApp/Call- 8596976190
Download “Tech Of World” App

14- Fill in the blanks with suitable Preposition from the given
alternatives.
The pen is _____________ the paper.
[OSSC IPO 2021]
(A) beneath
(B) since
(C) about
(D) till

ANS- A

The appropriate preposition to fill in the blank is (A) beneath. The


sentence would be: "The pen is beneath the paper." The preposition
"beneath" is used to indicate position or location below or underneath
something.

Example:

• The key was hidden beneath the doormat.


• The cat found a cozy spot beneath the blanket.

• He discovered a secret compartment beneath the floorboards.

Join Us-
ଓଡ଼ିଶାର ସମ� ପୂବର୍ ବଷର୍ର �ଶ� ଉ�ର ପାଇଁ
WtsApp/Call- 8596976190
Download “Tech Of World” App

15- Fill in the blanks with suitable Preposition from the given
alternatives.
A gauge of oil-market volatility ____________ Thursday rose to the
highest level since March of last year.
[Odisha Police ASI 2022]
(A) at
(B) through
(C) in
(D) on

ANS- D

The appropriate preposition to fill in the blank is (D) on. The sentence
would be: "A gauge of oil-market volatility on Thursday rose to the
highest level since March of last year." The preposition "on" is used to
indicate the specific day or time when something occurred.

Example:

• The concert will take place on Saturday evening.


• The meeting is scheduled for Tuesday morning.

• The report was released on June 1st.

Join Us-
ଓଡ଼ିଶାର ସମ� ପୂବର୍ ବଷର୍ର �ଶ� ଉ�ର ପାଇଁ
WtsApp/Call- 8596976190
Download “Tech Of World” App

16- Fill in the blanks with suitable Preposition from the given
alternatives.
The key chain is _____________ the paper.
[OSSC Traffic SI 2022 Exam]
(A) beneath
(B) since
(C) till
(D) about

ANS- A

The correct preposition to fill in the blank is "beneath." The sentence


should be "The key chain is beneath the paper." This preposition is used
to indicate that something is below or under another object.

Additional examples:

1. The cat hid beneath the bed.


2. I found my lost keys beneath a pile of clothes.

3. The treasure was buried beneath the old oak tree.

Join Us-
ଓଡ଼ିଶାର ସମ� ପୂବର୍ ବଷର୍ର �ଶ� ଉ�ର ପାଇଁ
WtsApp/Call- 8596976190
Download “Tech Of World” App

17- Fill in the blanks with correct option:


Deepa worked _______ great sincerity.
[Odisha High School Teacher Exam 2021]
A) for
B) in
C) with
D) by

ANS- C

The correct option to fill in the blank is "with." The sentence should be
"Deepa worked with great sincerity." This preposition is used to indicate
collaboration, association, or the manner in which an action is
performed.

Additional examples:

1. He painted the picture with a brush.


2. They worked together with dedication.

3. She spoke with confidence during the presentation.

Join Us-
ଓଡ଼ିଶାର ସମ� ପୂବର୍ ବଷର୍ର �ଶ� ଉ�ର ପାଇଁ
WtsApp/Call- 8596976190
Download “Tech Of World” App

18- Put the correct Preposition in the blank:


I am eager _________ see you.
[Odisha B.Ed Exam 2017]
(a) to
(b) for
(c) with
(d) at

ANS- A

Explanation: The correct preposition in this sentence is "to." When we


are eager, we have a strong desire or anticipation to do something.
Therefore, I am eager to see you.

Additional examples:
- He was eager to learn new skills and enrolled in a training program.
- The children were eager to open their presents on Christmas
morning.

Join Us-
ଓଡ଼ିଶାର ସମ� ପୂବର୍ ବଷର୍ର �ଶ� ଉ�ର ପାଇଁ
WtsApp/Call- 8596976190
Download “Tech Of World” App

19- Choose the correct option to join the two sentences:


There will be a test today, ________it was not announced
yesterday.
[OSSC Traffic Constable 2022 Exam]
A) even if
B) but also
C) as though
D) so that

ANS- A

The correct option to join the two sentences is "even if." The sentence
should be "There will be a test today, even if it was not announced
yesterday." "Even if" is used to express a hypothetical or uncertain
condition.

Additional examples:

1. I will go to the party even if I'm feeling tired.


2. She will take the job even if the salary is not very high.
3. We will continue the project even if we face challenges along the
way.

Join Us-
ଓଡ଼ିଶାର ସମ� ପୂବର୍ ବଷର୍ର �ଶ� ଉ�ର ପାଇଁ
WtsApp/Call- 8596976190
Download “Tech Of World” App

20- Choose the right option to fill the blank:


The school was situated on the opposite bank of the river and the
students had to swim __________ the river to get to the school.
[Odisha CT Exam 2018]
A) through
B) across
C) by
D) along

ANS- B

The correct answer is B) across. The sentence should be "The school


was situated on the opposite bank of the river and the students had to
swim across the river to get to the school." The word "across" indicates
the direction of movement from one side of the river to the other.

Additional examples:

1. We walked across the bridge to reach the other side.


2. They drove across the country to visit their relatives.
3. She ran across the field to catch the ball.

Join Us-
ଓଡ଼ିଶାର ସମ� ପୂବର୍ ବଷର୍ର �ଶ� ଉ�ର ପାଇଁ
WtsApp/Call- 8596976190
Download “Tech Of World” App

21- I think I'll wait ____________ Saturday before I decide what to


do.
[OPSC ASO Exam 2019]
(A) till
(B) before
(C) unto
(D) until

ANS- D

The correct answer is D) until. The sentence should be "I think I'll wait
until Saturday before I decide what to do." The word "until" indicates the
point in time up to which the action of waiting will continue.

Additional examples:

1. We will keep the secret until she returns.


2. Please wait until the traffic clears before crossing the street.
3. He couldn't wait until Christmas to open his presents.

Join Us-
ଓଡ଼ିଶାର ସମ� ପୂବର୍ ବଷର୍ର �ଶ� ଉ�ର ପାଇଁ
WtsApp/Call- 8596976190
Download “Tech Of World” App

22- Choose the correct Preposition:


The children skated _________ the hill.
[Odisha High School Teacher Exam 2022]
A) with
B) in
C) down
D) above

ANS- C

Explanation: The correct preposition in this sentence is "down." When we


skate, we typically move in a downward direction, especially when going
down a slope or hill. Therefore, the children skated down the hill.
Additional examples:
- The kids slid down the slide at the playground.
- The skiers raced down the mountain slope.

Join Us-
ଓଡ଼ିଶାର ସମ� ପୂବର୍ ବଷର୍ର �ଶ� ଉ�ର ପାଇଁ
WtsApp/Call- 8596976190
Download “Tech Of World” App

23- Fill in the blanks with suitable Preposition from the given
alternatives.
We have been living in this palace ___________ 1956.
[OSSC Excise SI]
(A) against
(B) since
(C) for
(D) through

ANS- B

Explanation: The correct preposition in this sentence is "since." We use


"since" to indicate the starting point of an action or event that continues
until the present. Therefore, we have been living in this palace since
1956.

Additional examples:

1. He has been working at the company since last year.


2. They have been friends since childhood.
3. She has been studying English since she was a teenager.

Join Us-
ଓଡ଼ିଶାର ସମ� ପୂବର୍ ବଷର୍ର �ଶ� ଉ�ର ପାଇଁ
WtsApp/Call- 8596976190
Download “Tech Of World” App

24- He couldn't keep himself from falling ________ the wall.


[Odisha Jail Warder 2022]
A. to
B. in
C. off
D. against

Ans- C
Explanation: The correct preposition in this sentence is "off." The
phrase "falling off the wall" indicates the movement or action of falling
away or down from the wall.
Examples:
1. The ball rolled off the table and onto the floor.
2. She accidentally dropped her phone off the balcony.
3. They climbed over the fence and jumped off the wall.

Join Us-
ଓଡ଼ିଶାର ସମ� ପୂବର୍ ବଷର୍ର �ଶ� ଉ�ର ପାଇଁ
WtsApp/Call- 8596976190
Download “Tech Of World” App

Topic Name- Speech


1- Rama said to me, "You will be rewarded".
[OSSSC RI Exam 2021]

(A) Rama told me that you will be rewarded.


(B) Rama told me that I would be rewarded.
(C) Rama told me that I will be rewarded.
(D) Rama told me that he will be rewarded.

Ans- B

The direct speech is "Rama said to me, 'You will be rewarded'". The reporting verb is
"said". The indirect speech is "Rama told me that I would be rewarded".

Explanation:

• The reporting verb is "said" in the past simple tense. So, the tense of the direct
speech must be changed to the past simple.
• The word "will" in the direct speech is changed to "would" in the indirect speech
because the reporting verb is in the past simple tense.
• The word "you" is changed to "me" because the speaker is the one being
rewarded.

Other options:

• Option (A) is incorrect because the word "you" is not changed to "me".
• Option (C) is incorrect because the word "will" is not changed to "would".
• Option (D) is incorrect because the word "he" is used instead of "I".

Join Us-
ଓଡ଼ିଶାର ସମ� ପୂବର୍ ବଷର୍ର �ଶ� ଉ�ର ପାଇଁ
WtsApp/Call- 8596976190
Download “Tech Of World” App

2- He said, "I saw her the day before yesterday".


(Choose the correct one in indirect speech.)
[OSSSC Junior Clerk 2018]

A) He said that he saw her the day before yesterday.


B) He said that he had seen her two days before.
C) He said that he had seen her one day before yesterday.
D) He said that he has seen her two days before yesterday.

Ans- B

The reporting verb in the direct speech is "said". This is in the past tense, so the tense
of the reported speech must be changed to the past perfect tense in the indirect
speech. Therefore, the correct indirect speech is "He said that he had seen her two
days before."

• Other Options:
• Option A is incorrect because the reporting verb "said" is in the past tense, but the
reported speech is in the simple past tense.
• Option C is incorrect because the reporting verb "said" is in the past tense, but the
reported speech is in the past perfect continuous tense.
• Option D is incorrect because the reporting verb "said" is in the past tense, but the
reported speech is in the present perfect tense.

Join Us-
ଓଡ଼ିଶାର ସମ� ପୂବର୍ ବଷର୍ର �ଶ� ଉ�ର ପାଇଁ
WtsApp/Call- 8596976190
Download “Tech Of World” App

3- He said, "I saw her the day before yesterday".


(Choose the correct one in indirect speech.)
[OSSSC Junior Clerk 2018]

A) He said that he saw her the day before yesterday.


B) He said that he had seen her two days before.
C) He said that he had seen her one day before yesterday.
D) He said that he has seen her two days before yesterday.

Ans- B

The reporting verb in the direct speech is "said". This is in the past tense, so the tense
of the reported speech must be changed to the past perfect tense in the indirect
speech. Therefore, the correct indirect speech is "He said that he had seen her two
days before."

• Other Options:
• Option A is incorrect because the reporting verb "said" is in the past tense, but the
reported speech is in the simple past tense.
• Option C is incorrect because the reporting verb "said" is in the past tense, but the
reported speech is in the past perfect continuous tense.
• Option D is incorrect because the reporting verb "said" is in the past tense, but the
reported speech is in the present perfect tense.

Join Us-
ଓଡ଼ିଶାର ସମ� ପୂବର୍ ବଷର୍ର �ଶ� ଉ�ର ପାଇଁ
WtsApp/Call- 8596976190
Download “Tech Of World” App

4- Convert the given direct/ indirect speech to indirect/ direct


speech with the help of the given options.
He said to her, "Why don't you go today?"
[OSSC Traffic SI 2022 Exam]

(A) He asked her why she did not go that day.


(B) He told her why she does not go that day.
(C) He asked her why she does not go that day.
(D) He told her why she did not go that day.

Ans- A

The correct option is A. The sentence in direct speech is "He said to her, "Why don't
you go today?"". The indirect speech of this sentence is "He asked her why she did not
go that day.".

• Explanation: In indirect speech, the reporting verb is "asked". The word "do" is
changed to "did" because the reported speech is in the past tense. The
sentence is not enclosed in quotation marks.
• Details of other options:
o Option B is incorrect because the word "tell" is used instead of "ask".
o Option C is incorrect because the word "does" is used instead of "did".
o Option D is incorrect because the sentence is not in indirect speech.

Join Us-
ଓଡ଼ିଶାର ସମ� ପୂବର୍ ବଷର୍ର �ଶ� ଉ�ର ପାଇଁ
WtsApp/Call- 8596976190
Download “Tech Of World” App

5- You will find a sentence using indirect speech (words actually


spoken that have been reported) followed by four options. You are
required to choose the one that is the correct form of the sentence
indirect speech (words actually spoken).
Donna greeted her professor in the morning.
[OSSC Junior Assistant 2022 Exam]

A) Donna is saying Good morning Professor


B) Donna said, "Good morning, Professor!"
C) Donna said that "Good morning, Professor!"
D) Donna had said that, "Good morning, Professor"

Ans- B

The sentence "Donna greeted her professor in the morning" is in indirect speech. This
means that the words are not being spoken directly, but are being reported by
someone else. In indirect speech, we do not use quotation marks. We also do not use
the reporting verb "said" in the present tense. Therefore, the correct option is "Donna
said, 'Good morning, Professor!'"

Other Options:

• Option (A): "Donna is saying Good morning Professor" This option is incorrect
because the words are not being spoken directly.
• Option (C): "Donna said that "Good morning, Professor!"" This option is incorrect
because we do not use quotation marks in indirect speech.
• Option (D): "Donna had said that, "Good morning, Professor"" This option is
incorrect because the reporting verb "said" is in the past tense, but the words in
the direct speech are in the present tense.

Join Us-
ଓଡ଼ିଶାର ସମ� ପୂବର୍ ବଷର୍ର �ଶ� ଉ�ର ପାଇଁ
WtsApp/Call- 8596976190
Download “Tech Of World” App

6- Convert the given direct/ indirect speech to indirect/ direct


speech with the help of the given options.
Karan said, “We are playing a game today."
[OSSC CGL Exam 2022]

(A) Karan said to that they were playing a game that day.
(B) Karan said that they were playing a game today.
(C) Karan said that they are playing a game that day.
(D) Karan said that they were playing a game that day.

Ans- D
• Explanation:

The reporting verb in the indirect speech is "said", which is in the past tense.
Therefore, the verb in the reported speech must also be in the past tense. The
original sentence is in the present tense, so we must change it to the past tense.
The word "we" is changed to "they" to agree with the subject of the reporting verb.

Other options:

o (A) Karan said to that they were playing a game that day. Incorrect. The
word "to" is not used correctly in the reported speech.
o (B) Karan said that they are playing a game that day. Incorrect. The verb in
the reported speech is in the present tense, even though the reporting verb
is in the past tense.
o (C) Karan said that they were playing a game today. Incorrect. The word
"today" is not changed to "that day" to agree with the tense of the reporting
verb.

Join Us-
ଓଡ଼ିଶାର ସମ� ପୂବର୍ ବଷର୍ର �ଶ� ଉ�ର ପାଇଁ
WtsApp/Call- 8596976190
Download “Tech Of World” App

Topic Name- Synonym


1- Choose the most appropriate similar meaning word:
IRONIC
[Odisha CT Exam 2019]

A) Inflexible
B) Good-natured
C) Disguisedly sarcastic
D) Bitter sweet

Ans- C
The most appropriate similar meaning word for "IRONIC" is (C)
Disguisedly sarcastic. Both words describe something that is contrary to
what is expected or has a hidden or mocking meaning.
A) Inflexible: Not easily bent, changed, or adapted; rigid or stubborn in
opinions or behavior.
B) Good-natured: Having a friendly, pleasant, or amiable disposition.
C) Disguisedly sarcastic: Concealing or hiding sarcasm behind a different
tone or expression.
D) Bittersweet: Evoking both positive and negative emotions or feelings;
having a mixture of happiness and sadness.

A) Inflexible: ସହଜେର ମୁ� ନୁ ଆଇ


ଁ ବା, ବଦଳିବା କିମ�ା ଅନୁ କୂଳ ନ େହବା; ମତାମତ କିମ�ା
ଆଚରଣେର କେଠାର କିମ�ା ଜି�ି ।

B) Good-natured: ବ�ୁ �ପୂ�ର୍, ଆନ�ଦାୟକ କିମ�ା ମି� �ଭାବ ରଖ�ୁ ।

C) Disguisedly sarcastic: ଏକ ଭି� �ର ବା ଅଭିବ୍ୟ�ି ପଛେର କଟା� ଲୁ ଚାଇବା ବା ଲୁ ଚାଇବା।

D) Bittersweet: ଉଭୟ ସକାରା�କ ଏବଂ ନକାରା�କ ଭାବନା ବା ଭାବନାକୁ ଉ�ାହିତ କରିବା; ସୁଖ
ଓ ଦୁ ଃଖର ମି�ଣ ରହିଛ।ି

Join Us-
ଓଡ଼ିଶାର ସମ� ପୂବର୍ ବଷର୍ର �ଶ� ଉ�ର ପାଇଁ
WtsApp/Call- 8596976190
Download “Tech Of World” App

2- Choose the word which best expresses the similar meaning of


the given word " SUBSTANTIAL ".
[OSSC Traffic SI 2022 Exam]

(A) Miniature
(B) Selfish
(C) Insignificant
(D) Important

Ans- D
The word that best expresses the similar meaning of "SUBSTANTIAL" is
"D) Important." "Substantial" and "Important" both refer to something
significant or of great importance.
(A) Miniature: A small-scale or reduced-size version of something; very
small in size or scale.
(B) Selfish: Concerned primarily with one's own interests, needs, or
welfare; lacking consideration for others.
(C) Insignificant: Of little importance or impact; not significant or
noteworthy.
(D) Important: Significant or of great value; having a significant impact or
influence.
(A) Miniature: େକୗଣସି ଜିନିଷର ଏକ �ୁ� େ�ଲ ବା କମ୍ ଆକାରର ସଂ�ରଣ; ଆକାର କି�ା�
େ�ଲେର ବହୁ ତ େଛାଟ ।
(B) Selfish: ମୁଖ�ତଃ ନିଜ �ାଥ�, ଆବଶ�କତା କି�ା� କଲ�ାଣ ବିଷୟେର ଚି�ିତ; ଅନ�ମାନ�
ପାଇ ଁ ବିଚାରର ଅଭାବ ।
(C) Insignificant: ଅ� ଗୁରୁ� ବା �ଭାବ; ଗୁରୁ�ପୂ�� କି�ା� ଉେ�ଖନୀୟ ନୁ େହ।ଁ
(D) Important: ଗୁରୁ�ପୂ�� କି�ା� ବହୁ ତ ମୂଲ�ବାନ; ଏହାର ଏକ ଗୁରୁ�ପୂ�� �ଭାବ ବା �ଭାବ
ରହିଛି ।

Join Us-
ଓଡ଼ିଶାର ସମ� ପୂବର୍ ବଷର୍ର �ଶ� ଉ�ର ପାଇଁ
WtsApp/Call- 8596976190
Download “Tech Of World” App

3- Choose the word which best expresses the similar meaning of


the given word " STAGGER ".
[Odisha Police ASI 2022]

(A) Shake
(B) Steady
(C) Peak
(D) Continue

Ans- A

The word that best expresses the similar meaning of "STAGGER" is "A)
Shake." "Stagger" means to walk or move unsteadily or in a shaky
manner.
(A) Shake: To tremble or vibrate rapidly; to cause to move or sway with
quick, short movements.
(B) Steady: Stable, constant, or unchanging; not easily disturbed or
disrupted.
(C) Peak: The highest point or level of something; the apex or
culmination.
(D) Continue: To persist or carry on without interruption; to proceed or
extend in time or space.
(A) Shake: �ତ ଗତିେର କ�ନ ବା କ�ନ କରି ବା; �ତ, �ୁ� ଗତିବିଧ � ସହିତ ଗତି କରି ବା
କି�ା� ଗତି କରି ବା ।
(B) Steady: �ିର, �ିର କି�ା� ଅପରି ବ�� ତ; ସହଜେର ବିଚଳିତ କି�ା� ବାଧା�ା� ହୁ ଏ ନାହିଁ।
(C) Peak: େକୗଣସି ଜିନିଷର ସେବ�ା� ବି�ୁ ବା �ର; ଶୀଷ� କି�ା� େଶଷ।
(D) Continue: ବିନା ବାଧାେର ଚାଲି ବା ବା ଜାରି ରଖ �ବା; ସମୟ କି�ା� �ାନେର ଆଗକୁ ବଢିବା
କି�ା� ବି�ାର କରି ବା ।

Join Us-
ଓଡ଼ିଶାର ସମ� ପୂବର୍ ବଷର୍ର �ଶ� ଉ�ର ପାଇଁ
WtsApp/Call- 8596976190
Download “Tech Of World” App

4- What is the synonym of "veracity"?


[OPSC ASO Exam 2022]

(A) Inaccuracy
(B) Reality
(C) Redundancy
(D) Falsehood

Ans- B

The synonym of "veracity" is "B) Reality." "Veracity" refers to the quality


of being truthful or honest, and "reality" carries a similar meaning.
(A) Inaccuracy: Lack of accuracy or precision; the state of being incorrect
or erroneous.
(B) Reality: The state or quality of being real or existing in fact; the truth
or actuality of something.
(C) Redundancy: The inclusion of unnecessary or repetitive elements;
the state of being excessive or superfluous.
(D) Falsehood: A statement or assertion that is not true; a lie or untruth.
(A) Inaccuracy: ସଠିକତା କି�ା� ସଠିକତାର ଅଭାବ; ଭୁଲ୍ ବା ଭୁଲ୍ େହବାର ଅବ�ା ।
(B) Reality: ବା�ବେର ବା�ବ ବା ବିଦ�ମାନ େହବାର ଅବ�ା ବା ଗୁଣବ�ା; େକୗଣସି ଜିନିଷର
ସତ�ତା ବା ବା�ବତା ।
(C) Redundancy: ଅନାବଶ�କ କି�ା� ପୁନରାବୃ �ି ଉପାଦାନର ଅ�ଭ�ୁ�ି ; ଅତ�ଧ �କ କି�ା�
ଅତ�ଧ �କ େହବାର ଅବ�ା ।
(D) Falsehood: ଏକ ବିବୃତି ବା ଦାବି ଯାହା ସତ ନୁ େହ;ଁ ମିଛ ନା ଅସତ� ।

Join Us-
ଓଡ଼ିଶାର ସମ� ପୂବର୍ ବଷର୍ର �ଶ� ଉ�ର ପାଇଁ
WtsApp/Call- 8596976190
Download “Tech Of World” App

5- What is the synonym of urge


[Odisha Police SI 2022 Exam]

A- disgust
B- deterrent
C- hindrance
D- Yearning

Ans- D

The synonym of "urge" is "D) Yearning." "Urge" refers to a strong desire


or impulse, and "yearning" conveys a similar meaning.
A- disgust: A strong feeling of revulsion, intense dislike, or repugnance
towards something.
B- deterrent: Something that discourages or prevents someone from
taking a particular action; a deterrent.
C- hindrance: An obstacle or impediment that hinders or obstructs
progress, movement, or achievement.
D- yearning: A strong desire or longing for something; a deep yearning or
craving.
A- disgust: େକୗଣସି ଜିନିଷ �ତି ତୀ� ଆେ�ାଶ, ତୀ� ନାପସ� କି�ା� ନି�ାର ତୀ� ଅନୁ ଭବ ।
B- deterrent: ଏପରି କିଛି ଯାହା କାହାକୁ ଏକ ନି��� କାଯ�� କରି ବାକୁ ନିରୁ�ାହିତ କେର କି�ା�
�ତିେରାଧ କେର; ଏକ �ତିବ�କ ।
C- hindrance: ଏକ �ତିବ�କ ବା �ତିବ�କ ଯାହା �ଗତି, ଗତିବିଧ � କି�ା� ସଫଳତାେର ବାଧା
ସୃ�ି କେର କି�ା� ବାଧା ସୃ�ି କେର ।
D- yearning: େକୗଣସି ଜିନିଷ ପାଇ ଁ ଦୃ ଢ଼ ଇ�ା ବା ଇ�ା; ଏକ ଗଭୀର ଇ�ା ବା େଲାଭ ।

Join Us-
ଓଡ଼ିଶାର ସମ� ପୂବର୍ ବଷର୍ର �ଶ� ଉ�ର ପାଇଁ
WtsApp/Call- 8596976190
Download “Tech Of World” App

6- Select the synonym for the word given below. Tendentiousness


[OSSC Traffic Constable 2022 Exam]
A) Objectivity
B) Neutrality
C) Propensity
D) Liberalism

Ans- C
The synonym for "Tendentiousness" is "C) Propensity."
"Tendentiousness" refers to having a tendency or bias, and "Propensity"
conveys a similar meaning.
A) Objectivity: The quality of being objective or unbiased; based on facts,
evidence, or external reality rather than personal feelings or opinions.
B) Neutrality: The state of being neutral or impartial; not favoring any
particular side or position.
C) Propensity: An inherent inclination or tendency towards a particular
behavior or characteristic; a natural disposition or proclivity.
D) Liberalism: A political ideology or philosophy that emphasizes
individual rights, freedom, and social progress; supporting social and
political reforms.
A) Objectivity: ନିରେପ� ବା ନିରେପ� େହବାର ଗୁଣ; ବ��ି ଗତ ଭାବନା କି�ା� ମତାମତ
ପରି ବେ�� ତଥ�, �ମାଣ କି�ା� ବାହ� ବା�ବତା ଉପେର ଆଧାରି ତ ।
B) Neutrality: ନିରେପ� ବା ନିରେପ� େହବାର ଅବ�ା; େକୗଣସି ନି��� ପା�� କି�ା�
ଁ ।
ପଦବୀକୁ ସମଥ�ନ କରୁନାହା�ି
C) Propensity: ଏକ ନି��� ଆଚରଣ କି�ା� େବ�ଶ�
ି � �ତି ଏକ ଅ�ନ�ହତ
ି �ବୃ �ି ବା �ବୃ �ି;
ି �ଭାବ ବା �ବୃ �ି ।
ଏକ �ାକୃତକ
D) Liberalism: ଏକ ରାଜେନ�ତକ
ି ଆଦଶ� ବା ଦଶ�ନ ଯାହା ବ��ି ଗତ ଅଧ �କାର, �ାଧୀନତା
ଏବଂ ସାମାଜିକ �ଗତି ଉପେର ଗୁରୁ� ଦିଏ; ସାମାଜିକ ଏବଂ ରାଜେନ�ତକ
ି ସଂ�ାରକୁ ସମଥ�ନ
କରି ବା ।

Join Us-
ଓଡ଼ିଶାର ସମ� ପୂବର୍ ବଷର୍ର �ଶ� ଉ�ର ପାଇଁ
WtsApp/Call- 8596976190
Download “Tech Of World” App

7- Identify the synonym of the italicised word:


Mohan is proud of his brother who is an agile and athletic person.
[OSSC Junior Assistant 2022 Exam]

A) Awkward
B) Inspiring
C) Active
D) Adaptive
Ans- C
The synonym of the word "agile" is C) Active. The sentence describes
Mohan's brother as an agile and athletic person, indicating that he is quick,
nimble, and physically active.
A) Awkward: Uncomfortable or clumsy in manner, movement, or behavior;
lacking ease or grace.
B) Inspiring: Arousing or stimulating positive feelings, enthusiasm, or
creativity; motivating or encouraging.
C) Active: Engaged in physical or mental activity; energetic or lively.
D) Adaptive: Able to adjust or change behavior, strategies, or attitudes to
suit new conditions or circumstances; flexible or responsive.
A) Awkward: ଆଚରଣ, ଗତିବଧି
ି କିମ�ା ଆଚରଣେର ଅସହଜ ବା ଅଡ଼ୁଆ; ସହଜତା କିମ�ା ଅନୁ �ହର
ଅଭାବ ।

B) Inspiring: ସକାରା�କ ଭାବନା, ଉ�ାହ କିମା� ସୃଜନଶୀଳତାକୁ ଉ�ାହିତ କରିବା କିମ�ା ଉ�ାହିତ
କରିବା; ଉ�ାହିତ କିମ�ା ଉ�ାହଜନକ ।

C) Active: ଶାରୀରିକ କିମ�ା ମାନସିକ କାଯର୍୍ୟେର ନିେୟାଜିତ; ଶ�ିଶାଳୀ କିମା� ଜୀବ� ।

D) Adaptive: ନୂ ତନ ପରି�ିତ ି କିମ�ା ପରି�ିତ ି ଅନୁ ଯାୟୀ ଆଚରଣ, ରଣନୀତି କିମ�ା ମେନାଭାବକୁ
ଆଡଜ� କିମା� ପରିବ�ର୍ନ କରିବାେର ସ�ମ; ନମନୀୟ କିମା� �ତି�ିୟାଶୀଳ।

Join Us-
ଓଡ଼ିଶାର ସମ� ପୂବର୍ ବଷର୍ର �ଶ� ଉ�ର ପାଇଁ
WtsApp/Call- 8596976190
Download “Tech Of World” App

8- Choose the most appropriate similar meaning word:


IMPROVEMENT
[Odisha B.Ed Exam 2019]
A) Preference
B) Hindrance
C) Enhancement
D) Deterioration
Ans- C
The most appropriate similar meaning word for "IMPROVEMENT" is (C)
Enhancement. Both words describe the act of making something better or
the state of being improved.
A) Preference: A greater liking or inclination for one option over others; a
choice based on personal preference or liking.
B) Hindrance: An obstacle, barrier, or impediment that hampers or delays
progress or success.
C) Enhancement: The act of improving or augmenting something; making
it better, stronger, or more valuable.
D) Deterioration: The process of becoming worse, less valuable, or of
lower quality; the decline or degradation of something.

A) Preference: େଗାଟିଏ ବିକ�କୁ ଅନ୍ୟ ବିକ� ଅେପ�ା ଅଧିକ ପସ� କିମା� ଆ�ହ; ବ୍ୟ�ିଗତ
ପସ� କିମା� ପସ� ଆଧାରେର ଏକ ଚୟନ ।

B) Hindrance: ଏକ �ତିବ�କ, �ତିବ�କ କିମା� �ତିବ�କ ଯାହା �ଗତି କିମ�ା ସଫଳତାେର ବାଧା
ସୃ�ି କେର କିମା� ବିଳମ� କେର ।

C) Enhancement: େକୗଣସି ଜିନଷ


ି େର ଉ�ତି ବା ଉ�ତି ଆଣିବା କାଯର୍୍ୟ; ଏହାକୁ ଉ�ମ, ଶ�ିଶାଳୀ
କିମା� ଅଧିକ ମୂଲ୍ୟବାନ କରିଥାଏ ।

D) Deterioration: ଖରାପ େହବାର ��ିୟା ଖରାପ, କମ୍ ମୂଲ୍ୟବାନ କିମା� ନିମ� ମାନର; େକୗଣସି
ି ର ଅବ�ୟ ବା ଅବ�ୟ ।
ଜିନଷ

Join Us-
ଓଡ଼ିଶାର ସମ� ପୂବର୍ ବଷର୍ର �ଶ� ଉ�ର ପାଇଁ
WtsApp/Call- 8596976190
Download “Tech Of World” App

To Continue- Click Here

Join Us-
ଓଡ଼ିଶାର ସମ� ପୂବର୍ ବଷର୍ର �ଶ� ଉ�ର ପାଇଁ WtsApp/Call- 8596976190
Download “Tech Of World” App

ALL ODISHA MATH PYQ


E- Book (PDF)
All Odisha Exam Previous Year Chapter Wise MATH
Question
(ODIA & ENGLISH Language)
[Exam- OSSSC, OSSC, OPSC, Police SI / Constable, Battalion,
Fireman, Jail Warder, B.ED, RHT, CT, JT, OSSTET, OAVS, OSSSC
(PEO, RI, ARI, JA, LSI, OFDC, Group C)]

Tech Of World APP

Features Of “MATH” Chapter Wise E-Book


Total PYQ 5,565

Total Topic 44 (With Answer Key)

Language Odia & English

Best For All Odisha Exam- OSSSC, OSSC, OPSC, Police SI


/ Constable, Battalion, Fireman, Jail Warder,
B.ED, RHT, CT, JT, OSSTET, OAVS, OSSSC (PEO,
RI, ARI, JA, LSI, OFDC, Group C) & Other Exams
Download App To Click Here
Buy E-Book
Call / WhatsApp 8596976190

Click Here-

Join Us-
ଓଡ଼ିଶାର ସମ� ପୂବର୍ ବଷର୍ର �ଶ� ଉ�ର ପାଇଁ WtsApp/Call- 8596976190
Download “Tech Of World” App

Sl. No. Topic No. Of MCQ

1 Average 353
2 Percentage 369
3 Partnership 126
4 Mixture & Allegation 170
5 Ratio & Proportion 274
6 Time & Work 301
7 Compound Interest 152
8 Profit & Loss 225
9 Simple Interest 158
10 Speed, Time & Distance 218
11 Simplification 297
12 Data Interpretation 383
13 Problems on Trains 54
14 Boat & Stream 42
15 Tax 53
16 Decimal & Fractions 53
17 Pipes and Cisterns 46
18 Discount 59
19 Age Problem 82
20 Divisibility 212
Teaching Exam PYQ
21 Geometry 118
22 Mensuration- Volume and Surface Area 303
23 Logarithm 24
24 Probability 193
25 Statistic 184
26 Algebra , Polynomial & Quadratic 276
27 Relations & Functions 38
28 Square & Square Root 97
29 Number System 180
30 Miscellaneous 68
31 Permutation & Combination 12
32 Sets & Union 78
33 Trigonometry 65
34 Hight & Distance 11
35 HCF & LCM 135
36 Analytical Geometry 32
37 Arithmatic Problem PYQ 18
38 Calcuclus 13

Join Us-
ଓଡ଼ିଶାର ସମ� ପୂବର୍ ବଷର୍ର �ଶ� ଉ�ର ପାଇଁ WtsApp/Call- 8596976190
Download “Tech Of World” App

39 Coordinate Geometry 35
40 Determinant & Matrix 12
41 Differentiation & Integration 12
42 Limits & Continuity 9
43 Sequences and Series 15
44 Math Pedagogy 10
Total 5,565

Join Us-
ଓଡ଼ିଶାର ସମ� ପୂବର୍ ବଷର୍ର �ଶ� ଉ�ର ପାଇଁ WtsApp/Call- 8596976190
Download “Tech Of World” App

Topic Name- Percentage

1- What percent of 1692 is 423?

1692 ର େକେତ ଶତକଡା 423 ଅେଟ?

(A) 35
(B) 20
(C) 30
(D) 25
[OSSC RHT High School Teacher 2023]

Ans- D

2- My income is Rs 1200 per month. If I save 20% of it every month,


how much I shall save in a year?

- େମାର େରାଜଗାର ମାସି କ ୧୨୦୦ ଟ�ା । ଯଦି ମ ଁୁ �ତି ମାସେର ଏହାର ୨୦% ସ�ୟ
କେର, େତେବ ମ ଁୁ ବଷ�କୁ େକେତ ସ�ୟ କରି ବ?
ି

A. Rs 240
B. Rs 2880
C. Rs 1440
D. Rs 120
[Odisha Jail Warder 2022]

Ans- B

Join Us-
ଓଡ଼ିଶାର ସମ� ପୂବର୍ ବଷର୍ର �ଶ� ଉ�ର ପାଇଁ WtsApp/Call- 8596976190
Download “Tech Of World” App

3- If the given two numbers are respectively 6% and 24% of a third


number, then what percentage is the first number of the second
number

- ଯଦି ଦିଆଯାଇଥ�ବା ଦୁ ଇଟି ସଂଖ�ା ତୃ ତୀୟ ସଂଖ�ାର ଯଥା�େମ 6% ଏବଂ 24%


ଅେଟ, େତେବ ଦି�ତୀୟ ସଂଖ�ାର �ଥମ ସଂଖ�ା େକେତ �ତିଶତ ଅେଟ

(A) 15%
(B) 20%
(C) 25%
(D) None of these
[OSSSC Livestock Inspector 2021]

Ans- C

4- If a number is increased by 25% and then decreased by 25%, find


out the percentage of increase/decrease of final value of the
number.

- ଯଦି େକୗଣସି ସଂଖ�ା 25% ବୃ�ି କରାଯାଏ ଏବଂ ପେର 25% �ାସ କରାଯାଏ, େତେବ
ସଂଖ�ାର ଚୂ ଡ଼ା� ମୂଲ�ର ବୃ�ି/�ାସର �ତିଶତ ଜାଣ�।

(A) Increase by 6.25 %


(B) Decrease by 6.25%
(C) Increase by 9.25 %
(D) Decrease by 9.25 %
[OSSSC RI Exam 2021]

Ans- B

Join Us-
ଓଡ଼ିଶାର ସମ� ପୂବର୍ ବଷର୍ର �ଶ� ଉ�ର ପାଇଁ WtsApp/Call- 8596976190
Download “Tech Of World” App

5- In an examination, there were 600 boys and 400 girls. 60% of


boys and 50% of girls passed. The percentage of candidates that
failed is:

- େଗାଟିଏ ପରୀ�ାେର ୬୦୦ ବାଳକ ଓ ୪୦୦ ଝିଅ ଥ�େଲ। ୬୦% ପୁଅ ଓ ୫୦% ଝିଅ
ପାସ୍ କରି ଛ�ି। େଫଲ୍ େହାଇଥ�ବା �ାଥ�ୀ� �ତିଶତ େହଉଛି:

(A) 50%
(B) 55%
(C) 40%
(D) 44%
[OSSSC Combined Recruitment 2022]

Ans- D

6- 2600 candidates appeared for an examination, of which 2184


passed. Find the pass percentage.

୨୬୦୦ ପରୀ�ାଥ�ୀ ପରୀ�ା େଦଇଥ�ବା େବେଳ େସମାନ� ମ�ରୁ ୨୧୮୪ ଜଣ ପାସ୍


କରି ଛ�ି। ପାସ୍ �ତିଶତ ନି�ା� ରଣ କର�।

(A) 92%
(B) 80%
(C) 84%
(D) 88%
[Odisha Police ASI 2022]

Ans- C

Join Us-
ଓଡ଼ିଶାର ସମ� ପୂବର୍ ବଷର୍ର �ଶ� ଉ�ର ପାଇଁ WtsApp/Call- 8596976190
Download “Tech Of World” App

7- What percent of 11.4 kg is 3078 gms?

- 11.4 କିେଲା�ାମର େକେତ �ତିଶତ 3078 �ାମ?

(A) 29
(B) 23
(C) 27
(D) 25
[OSSC Traffic SI 2022 Exam]

Ans- C

8- The salary of X, Y and Z are in the ratio 5:3: 1. The salary of X and
Y together is Rs 7200. By what percent is salary of X more than that
of Z?

- X, Y ଏବଂ Z ର ଦରମା 5: 3: 1 ଅନୁ ପାତେର ଅଛି । X ଏବଂ Y ମିଳିତ ଦରମା େହଉଛି


୭୨୦୦ ଟ�ା । Z ତୁ ଳନାେର X ର ଦରମା େକେତ �ତିଶତ ଅଧ�କ?

(A) 100%
(B) 200%
(C) 300%
(D) 400%
[OSSSC RI Exam 2021]
Ans- D

Join Us-
ଓଡ଼ିଶାର ସମ� ପୂବର୍ ବଷର୍ର �ଶ� ଉ�ର ପାଇଁ WtsApp/Call- 8596976190
Download “Tech Of World” App

9- Rakesh spends 15% of his monthly income on his household


expenditure, 20% on books, 30% on clothes and saves the rest. On
counting, he comes to know that he has finally saved Rs.12565.
Find his monthly income. (In Rs.)

- ରାେକଶ ତା� ମାସି କ ଆୟର ୧୫% ଘର ଖ��, ୨୦% ବହି, ୩୦% େପାଷାକ ପାଇ ଁ ଖ��
କର�ି ଏବଂ ବାକି ସ�ୟ କର�ି । ଗଣନା କରି ବା ପେର, େସ ଜାଣିବାକୁ ପାଇେଲ େଯ େସ
େଶଷେର 12565 ଟ�ା ସ�ୟ କରି ଛ�ି। ତା�ର ମାସି କ ଆୟ ସ�ାନ କର�। (ଟ�ାେର)

(A) 35900
(B) 36900
(C) 37900
(D) 34900
[OSSC CGL Exam 2022]

Ans- A

10- After 5060 liters of petrol were poured into the tank, it was still
12% empty. What is the total capacity of the tank? (In liters)

- ଟା�ି େର ୫୦୬୦ ଲି ଟର େପେ�ାଲ ଢାଳିବା ପେର ମ� ତାହା ୧୨ �ତିଶତ ଖାଲି


ରହିଥିଲା। ଟା�ି ର ସମୁଦାୟ �ମତା େକେତ? (ଲି ଟରେର)

(A) 5750
(B) 5550
(C) 5450
(D) 5650
[OSSC Excise SI 2021]

Ans- A

Join Us-
ଓଡ଼ିଶାର ସମ� ପୂବର୍ ବଷର୍ର �ଶ� ଉ�ର ପାଇଁ WtsApp/Call- 8596976190
Download “Tech Of World” App

11- If 25% of a certain number is 1875, then what is 80% of that


number?

ଯଦି ଏକ ନି��� ସଂଖ�ାର ୨୫% ୧୮୭୫, େତେବ େସହି ସଂଖ�ାର ୮୦% େକେତ?

(A) 6000
(B) 6200
(C) 6100
(D) 6300
[OSSC SCEW 2022]

Ans- A

12. Ram secures 50% and Shyam secures 30% of the total marks in
an examination. If the difference between the marks secured by
both of them is 40, what is the total marks ?

ରାମ ଏକ ପରୀ�ାେର େମାଟ ମାକ�ର ୫୦% ଏବଂ ଶ�ାମ ୩୦% ମାକ� ହାସଲ କର�ି ।
ଯଦି ଦୁ େହ ଁ ହାସଲ କରି ଥବ
� ା ମାକ� ମ�େର ପାଥ�କ� ୪୦, େତେବ ସମୁଦାୟ ମାକ� େକେତ
?
(A) 100
(B) 500
(C) 300
(D) 200
[Odisha District Police Constable 2013]

Ans- D

To Continue- Click Here

Join Us-
ଓଡ଼ିଶାର ସମ� ପୂବର୍ ବଷର୍ର �ଶ� ଉ�ର ପାଇଁ WtsApp/Call- 8596976190
Download “Tech Of World” App

ALL ODISHA PEDAGOGY PYQ


E- Book (PDF)
All Odisha Exam Previous Year Chapter Wise
PEDAGOGY (Teaching Aptitude) Question
[Exam- B.ED, RHT, CT, JT, OSSTET, OTET, OAVS]

Tech Of World APP

Features Of “PEDAGOGY (Teaching Aptitude)” Chapter


Wise E-Book
Total PYQ 3,391

Language Odia & English

Explanation Odia & English

Best For All Odisha Exam- [Exam- B.ED, RHT, CT, JT,
OSSTET, OTET, OAVS]
Download App Click Here

Contact 8596976190

Click Here

Join Us-
ଓଡ଼ିଶାର ସମ� ପୂବର୍ ବଷର୍ର �ଶ� ଉ�ର ପାଇଁ WtsApp/Call- 8596976190
Download “Tech Of World” App

Odisha Teaching Exams Previous Year Questions (PYQ)


Sl. No. Exam Name Year
1 CT 2018 (All 23 Shifts)
2019 (All 24 Shifts)
2020 (All 20 Shifts)
2023 (All 11 Shifts)
2 OTET 2022
3 JT 2023 (All 14 Shifts)
4 B.ED Entrance Exam
ARTS & SCIENCE 2018 (All 11 Shifts)
2019 (All 15 Shifts)
2020 (All 13 Shifts)
2021 (All 15 Shifts)
2022
2023
6 OSSTET (ARTS, CBZ & PCM) 2018, 2019, 2021 (1st),
2021 (2nd ), 2022, 2024
7 RHT / High School Teacher
 RHT ARTS 2019
2021 (All 3 Shifts)
2022 (All 6 Shifts)
2023 Pre (All 3 Shifts)
2023 Mains
 RHT CBZ 2019
2021
2022 (All 3 Shifts)
2023 Pre (All 3 Shifts)
2023 Mains
 RHT PCM 2019 (All 2 Shifts)
2021 (All 2 Shifts)
2022 (All 4 Shifts)
2023 Pre (All 3 Shifts)
2023 Mains
8 OAVS
OAVS TGT ODIA 2018, 2019, 2021
OAVS TGT ENGLISH 2019
OAVS TGT SOCIAL STUDIES 2019, 2021, 2023
OAVS TGT SCIENCE 2019, 2021
OAVS TGT MATH 2019, 2023

Join Us-
ଓଡ଼ିଶାର ସମ� ପୂବର୍ ବଷର୍ର �ଶ� ଉ�ର ପାଇଁ WtsApp/Call- 8596976190
Download “Tech Of World” App

1- Clarifying relationships with diagrams or graphs or clarifying


processes with flow charts is done in which method of teaching?
ଡାଏ�ାମ କି�ା� �ାଫ୍ ସହିତ ସ�କ� �� କରି ବା କି�ା� େ�� ା ଚାଟ� ସହିତ ��ିୟାକୁ ��
କରି ବା ଶି�ାଦାନର େକଉ ଁ ପ�ତିେର କରାଯାଏ?

A) Lecture method
B) Immersive environments
C) Graphic organizing
D) Interactive teaching
[Odisha High School Teacher Exam 2019]

Ans- C
• Short Explanation: Graphic organizers are visual representations of
information that can help students to understand and remember complex
concepts. They can be used to clarify relationships between different ideas, as
well as to illustrate processes and procedures.
• Short Explanation of Other Options:
o A. Lecture method: This is a traditional teaching method in which the
teacher delivers information to the students in a one-way format.
o B. Immersive environments: These are learning environments that are
designed to simulate real-world situations.
o D. Interactive teaching: This is a teaching approach that encourages
students to actively engage with the learning material.

Join Us-
ଓଡ଼ିଶାର ସମ� ପୂବର୍ ବଷର୍ର �ଶ� ଉ�ର ପାଇଁ WtsApp/Call- 8596976190
Download “Tech Of World” App

• ସଂ�ି� ବିବରଣୀ: �ାଫିକ୍ ଆେୟାଜକେହଉଛି ସୂଚନାର ଦୃ ଶ� ଉପ�ାପନା ଯାହା ଛା�ଛା�ୀମାନ�ୁ


ଜଟିଳ ଧାରଣାକୁ ବୁ ଝିବା ଏବଂ ମେନ ରଖ�ବାେର ସାହାଯ� କରି ପାେର । େସମାେନ ବିଭି� ଧାରଣା ମ�େର
ସ�କ� କୁ �� କରି ବା ସହିତ ��ିୟା ଏବଂ ��ିୟାଗୁଡ଼ିକୁ ବ��ନା କରି ବା ପାଇ ଁ ବ�ବହୃ ତ େହାଇପାର�ି |
• ଅନ� ବିକ�ଗୁଡ଼ିକର ସଂ�ି� ବିବରଣୀ:
o A. ବକ୍ତୃତା ପ�ତି: ଏହା ଏକ ପାର�ରି କ ଶି�ାଦାନ ପ�ତି େଯଉଥଁ �େର ଶି�କ ଏକତରଫା
ଫମ�ାଟେର ଛା�ଛା�ୀ�ୁ ସୂଚନା �ଦାନ କର�ି ।
o B. ଇମସ�ଭ୍ ପରି େବଶ: ଏଗୁଡ଼ିକ ଶି�ଣ ପରି େବଶ ଯାହା ବା�ବ ଦୁ ନିଆର ପରି �ିତିକୁ ଅନୁ କରଣ
ଁ ଜାଇନ୍ କରାଯାଇଛି ।
କରି ବା ପାଇ ଡି
o D. ଇ�ରଆ�ି�ଭ୍ ଶି�ାଦାନ: ଏହା ଏକ ଶି�ଣ ପ�ତି ଯାହା ଛା�ମାନ�ୁ ଶି�ଣ ସାମ�ୀ ସହିତ ସ�ିୟ
ଭାବେର ଜଡ଼ିତ େହବାକୁ ଉ�ାହିତ କେର |

2- The concept of micro teaching was developed in


ମାଇେ�ା ଟିଚିଂର ଧାରଣା ବିକଶିତ େହାଇଥ �ଲା

A) Oxford University
B) Cambridge University
C) Princeton University
D) Stanford University
[Odisha High School Teacher Exam 2019]

Ans- D

Join Us-
ଓଡ଼ିଶାର ସମ� ପୂବର୍ ବଷର୍ର �ଶ� ଉ�ର ପାଇଁ WtsApp/Call- 8596976190
Download “Tech Of World” App

• Short Explanation: Microteaching was developed in the early 1960s at Stanford


University by Allen Corwin and Ned Flanders. It is a teacher training technique
that involves breaking down teaching tasks into smaller, more manageable
units. This allows teachers to practice and receive feedback on their teaching
skills in a controlled setting.
• Short Explanation of Other Options:
o A. Oxford University: This is a prestigious university in the United
Kingdom.
o B. Cambridge University: This is another prestigious university in the
United Kingdom.
o C. Princeton University: This is a prestigious university in the United
States.
• ସଂ�ି� ବିବରଣୀ: ୧୯୬୦ ଦଶକର �ାର�େର �ାନେଫାଡ� ବି�ବିଦ�ାଳୟେର ଆେଲନ୍ େକାୱ�ନ୍ ଏବଂ
େନଡ୍ �� ା�ସ� � �ାରା ମାଇେ�ାଟିଚିଂ ବିକଶିତ େହାଇଥ�ଲା । ଏହା େହଉଛି ଏକ ଶି�କ �ଶି�ଣ େକୗଶଳ
େଯଉଥ�ଁ େର ଶି�ାଦାନ କାଯ��ଗୁଡ଼ିକୁ େଛାଟ, ଅଧକ� ପରି ଚାଳନାେଯାଗ� ୟୁନିଟେର ବିଭ� କରି ବା ଅ�ଭ�ୁ�
। ଏହା �ାରା ଶି�କମାେନ ଏକ ନିୟ�ିତ େସଟିଂେର େସମାନ� ଶି�ାଦାନ ଦ�ତା ଉପେର ଅଭ�ାସ ଏବଂ
ମତାମତ ପାଇପାରି େବ ।
• ଅନ� ବିକ�ଗୁଡ଼ିକର ସଂ�ି� ବିବରଣୀ:
o ଅ�େଫାଡ� ୟୁ ନିଭସ� ଟି: ଏହା �ିେଟନର ଏକ ସ�ାନଜନକ ବି�ବିଦ�ାଳୟ ।
o େକ�ି ��ଜ୍ ବି�ବିଦ�ାଳୟ: ଏହା �ିେଟନର ଅନ� ଏକ ସ�ାନଜନକ ବି�ବିଦ�ାଳୟ ।
o �ି�ଟ
� ନ୍ ବି�ବିଦ�ାଳୟ: ଏହା ଯୁ�ରା�� ଆେମରି କାର ଏକ ସ�ାନଜନକ ବି�ବିଦ�ାଳୟ ।

3- What are listening, reading, speaking and writing Skills referred


to in the teaching of English?
ଇଂରାଜୀ ଶି�ାେର କ'ଣ ଶୁଣବ
ି ା, ପଢ଼ିବା, କହିବା ଓ େଲଖ �ବାର ଦ�ତା ରହିଛି?

A) Objectives
B) Duties
C) Subjects
D) Skills
[Odisha High School Teacher Exam 2019]

Ans- D

Join Us-
ଓଡ଼ିଶାର ସମ� ପୂବର୍ ବଷର୍ର �ଶ� ଉ�ର ପାଇଁ WtsApp/Call- 8596976190
Download “Tech Of World” App

• Short Explanation: Listening, reading, speaking, and writing are the four
fundamental language skills. They are essential for communicating effectively in
any language.
• Short Explanation of Other Options:
o A.Objectives: These are statements that describe the desired outcomes of
instruction.
o B. Duties: These are the tasks and responsibilities that a teacher has.
o C.Subjects: These are the different areas of study that are taught in
schools.
• ସଂ�ି� ବିବରଣୀ: ଶୁଣବି ା, ପଢ଼ିବା, କହିବା ଏବଂ େଲଖ�ବା େହଉଛି ଚାରି ଟି େମୗଳିକ ଭାଷା ଦ�ତା
। େଯେକୗଣସି ଭାଷାେର �ଭାବଶାଳୀ ଭାବେର େଯାଗାେଯାଗ କରି ବା ପାଇ ଁ େସଗୁଡିକ ଜରୁରୀ ଅେଟ ।
• ଅନ� ବିକ�ଗୁଡ଼ିକର ସଂ�ି� ବିବରଣୀ:
o A. ଉେ�ଶ�: ଏଗୁଡ଼ିକ େହଉଛି ବିବୃତି ଯାହା ଶି�ାର ଇ�ିତ ଫଳାଫଳକୁ ବ��ନା କେର ।
o B. କ��ବ�: ଜେଣ ଶି�କ�ର ଏସବୁ କାଯ�� ଓ ଦାୟି � ରହିଛି।
o C. ବିଷୟ: ଏଗୁଡ଼ିକ େହଉଛି ଅ�ୟନର ବିଭି� େ�� ଯାହା ବିଦ�ାଳୟେର ପଢ଼ାଯାଏ ।

4- Which of the following is a reason for the popularity of learning


English in 21st Century?
ଏକବିଂଶ ଶତା�ୀେର ଇଂରାଜୀ ଶିଖବ
� ାର େଲାକ�ିୟତାର କାରଣ ନି�ଲ
� ି ଖତ
� ମ�ରୁ
େକଉଟିଁ ?

A) English is easier to learn than the other languages


B) Growing information technology
C) English is the only language taught in many schools
D) All countries have made it compulsory to learn english
[Odisha High School Teacher Exam 2022]

Ans- B

Join Us-
ଓଡ଼ିଶାର ସମ� ପୂବର୍ ବଷର୍ର �ଶ� ଉ�ର ପାଇଁ WtsApp/Call- 8596976190
Download “Tech Of World” App

• Short Explanation: English is the dominant language of information


technology, including programming languages, software, and the internet. As a
result, learning English is essential for accessing and participating in the global
information economy.
• Short Explanation of Other Options:
o A. English is easier to learn than the other languages: This is a
subjective statement and there is no consensus on whether English is
easier or harder to learn than other languages.
o C. English is the only language taught in many schools: While English
is widely taught in schools around the world, it is not the only language
taught.
o D. All countries have made it compulsory to learn English: This is not
true. Not all countries have made it compulsory to learn English.

• ସଂ�ି� ବିବରଣୀ: େ�ା�ାମିଂ ଭାଷା, ସ�� େୱୟାର୍ ଏବଂ ଇ�ରେନଟ୍ ସେମତ ସୂଚନା �ଯୁ�ିବିଦ�ାର
�ମୁଖ ଭାଷା େହଉଛି ଇଂରାଜୀ । ଫଳ�ରୂପ, ବି� ସୂଚନା ଅଥ�ନୀତିେର �େବଶ ଏବଂ ଅଂଶ�ହଣ କରି ବା
ପାଇ ଁ ଇଂରାଜୀ ଶିଖବ
� ା ଜରୁରୀ ଅେଟ ।
• ଅନ� ବିକ�ଗୁଡ଼ିକର ସଂ�ି� ବିବରଣୀ:
o A. ଅନ� ଭାଷା ତୁ ଳନାେର ଇଂରାଜୀ ଶିଖବ � ା ସହଜ: ଏହା ଏକ ବ��ି ଗତ ବିବୃତି ଏବଂ ଅନ�
� ା ସହଜ ନା କ�କର େସ ବିଷୟେର େକୗଣସି ସହମତି ନାହିଁ ।
ଭାଷା ତୁ ଳନାେର ଇଂରାଜୀ ଶିଖବ
o B. ଇଂରାଜୀ େହଉଛି ଏକମା� ଭାଷା ଯାହା ଅେନକ ବିଦ�ାଳୟେର ପଢ଼ାଯାଏ: ସାରା ବି�ର
ବିଦ�ାଳୟେର ଇଂରାଜୀ ବ�ାପକ ଭାବେର ପଢ଼ାଯାଏ, କି� ଏହା ଏକମା� ଭାଷା ନୁ େହ ଁ।

o D. ସବୁ େଦଶ ଇଂରାଜୀ ଶିଖିବା ବାଧ୍ୟତାମୂଳକ କରିଛ�ି: ଏହା ସତ ନୁ େହ।ଁ ସବୁ େଦଶ ଇଂରାଜୀ
ଶିଖବ
� ା ବା�ତାମୂଳକ କରି ନାହା�ି।

Join Us-
ଓଡ଼ିଶାର ସମ� ପୂବର୍ ବଷର୍ର �ଶ� ଉ�ର ପାଇଁ WtsApp/Call- 8596976190
Download “Tech Of World” App

5- In Odisha English is given which of the following status?


ଓଡ଼ିଶାେର ଇଂରାଜୀକୁ ନି�ଲ
� ି ଖତ
� ମ�ରୁ େକଉ ଁମାନ�ତା ଦିଆଯାଏ ?

A) L1
B) L3
C) L4
D) L2
[Odisha High School Teacher Exam 2022]

Ans- D

• Short Explanation: L1 refers to a person's first language, L2 refers to a


person's second language, and so on. In Odisha, English is typically taught as a
second language, so it is considered to be L2.
• Short Explanation of Other Options:

L1: This refers to a person's first language Odia.


• ସଂ�ି� ବିବରଣୀ: L1 ଜେଣ ବ��ି ର �ଥମ ଭାଷାକୁ ବୁ ଝାଏ, L2 ଜେଣ ବ��ି ର �ିତୀୟ ଭାଷାକୁ ବୁ ଝାଏ,
ଇତ�ାଦି । ଓଡ଼ିଶାେର ସାଧାରଣତଃ ଇଂରାଜୀକୁ �ିତୀୟ ଭାଷା ଭାେବ ପଢ଼ାଯାଏ, େତଣୁ ଏହାକୁ ଏଲ୍ -୨
େବାଲି ବିେବଚନା କରାଯାଏ।
• ଅନ� ବିକ�ଗୁଡ଼ିକର ସଂ�ି� ବିବରଣୀ:
• ଏଲ୍ ୧: ଏହା ଜେଣ ବ��ି ର �ଥମ ଭାଷା ଓଡ଼ିଆକୁ ବୁ ଝାଏ।

Join Us-
ଓଡ଼ିଶାର ସମ� ପୂବର୍ ବଷର୍ର �ଶ� ଉ�ର ପାଇଁ WtsApp/Call- 8596976190
Download “Tech Of World” App

6- Teaching English language is considered as a challenging task


because
1) conventional methods take maximum time
2) drilling, dealing with grammar and pronunciation take less time
ଇଂରାଜୀ ଭାଷା ଶିଖାଇବା ଏକ ଚ�ାେଲ�ି ଂ କାଯ�� େବାଲି ବିେବଚନା କରାଯାଏ କାରଣ
୧) ପାର�ରି କ ପ�ତିେର ସବ�ାଧ �କ ସମୟ ଲାଗି ଥାଏ

୨) ଖନନ, ବ�ାକରଣ ଓ ଉ�ାରଣ ସହ ମୁକାବିଲା କରି ବାକୁ କମ୍ ସମୟ ଲାଗି ଥାଏ

A) Only 2
B) Only 1
C) Both 1 & 2
D) Neither 1 nor 2
[Odisha High School Teacher Exam 2022]

Ans- B

• Short Explanation: Conventional teaching methods of English can take a lot


of time, especially when it comes to drilling, grammar, and
pronunciation. Newer teaching methods, such as communicative language
teaching (CLT), focus on creating opportunities for students to practice using
English in real-world contexts. These methods can be more effective and less
time-consuming.
• ସଂ�ି� ବିବରଣୀ: ଇଂରାଜୀର ପାର�ରି କ ଶି�ାଦାନ ପ�ତି େର ବହୁ ତ ସମୟ ଲାଗି ପାେର,
ବିେଶଷକରି େଯେତେବେଳ �ିଲିଂ, ବ�ାକରଣ ଏବଂ ଉ�ାରଣ ବିଷୟେର ଆେସ । ନୂ ତନ ଶି�ାଦାନ
ପ�ତି, େଯପରି କି େଯାଗାେଯାଗ ଭାଷା ଶି�ା (ସି ଏଲଟି), ବା�ବ ବି� ପରି େ��ୀେର ଇଂରାଜୀ
ବ�ବହାର କରି ବାକୁ ଛା�ଛା�ୀ� ପାଇ ଁ ସୁେଯାଗ ସୃ�ି କରି ବା ଉପେର �ାନ ଦିଅ�ି । ଏହି ପ�ତି
ଅଧକ� �ଭାବଶାଳୀ ଏବଂ କମ୍ ସମୟ ସାେପ� େହାଇପାେର |

Join Us-
ଓଡ଼ିଶାର ସମ� ପୂବର୍ ବଷର୍ର �ଶ� ଉ�ର ପାଇଁ WtsApp/Call- 8596976190
Download “Tech Of World” App

7- Which of the following options is FALSE with reference to the


aims of education in India as formulated by Secondary Education
Commission in 1952-53?
୧୯୫୨-୫୩ ମସି ହାେର ମା�ମିକ ଶି�ା ଆେୟାଗ �ାରା �ଣୟନ କରାଯାଇଥ�ବା
ଭାରତେର ଶି�ାର ଲ�� ସ���େର ନି�ଲ
� ି ଖତ
� ମ�ରୁ େକଉ ଁବିକ� ଭୁଲ୍ ଅେଟ?

A) Development of Democratic Citizenship


B) Improvement in Adult Female Literacy Rate
C) Improvement of Vocational Efficiency
D) Development of Education for Leadership
[Odisha CT Exam 2019]

Ans- B
• Short Explanation: The Secondary Education Commission of 1952-53 did not
specifically mention the improvement in adult female literacy rate as one of its
aims. However, the commission did recommend that the government take steps
to promote universal education for all children, regardless of gender.
• Short Explanation of Other Options:
o A. Development of Democratic Citizenship: This was one of the key
aims of the Secondary Education Commission. The commission believed
that education should help students to develop the values and skills
necessary to be responsible citizens in a democracy.
o C. Improvement of Vocational Efficiency: The commission also
recommended that secondary education should provide students with the
opportunity to develop vocational skills. This would help them to prepare
for employment and contribute to the economic development of the
country.
o D. Development of Education for Leadership: The commission
believed that education should help students to develop the leadership
qualities necessary to guide the country into the future.

Join Us-
ଓଡ଼ିଶାର ସମ� ପୂବର୍ ବଷର୍ର �ଶ� ଉ�ର ପାଇଁ WtsApp/Call- 8596976190
Download “Tech Of World” App

• ସଂ�ି� ବିବରଣୀ: ୧୯୫୨-୫୩ ର ମା�ମି କ ଶି�ା ଆେୟାଗ ଏହାର ଅନ�ତମ ଉେ�ଶ� ଭାବେର �ା�
ବୟ� ମହିଳା ସା�ରତା ହାରେର ଉ�ତିକୁ ନି��� ଭାବେର ଉେ�ଖ କରି ନଥ�େଲ । େତେବ ଲି �
ନିବ�େଶଷେର ସମ� ପିଲା� ପାଇ ଁ ସାବ�ଜନୀନ ଶି�ାକୁ େ�ା�ାହିତ କରି ବା ପାଇ ଁ ସରକାର ପଦେ�ପ
େନବାକୁ କମିଶନ ସୁପାରି ସ କରି ଛ�ି।
• ଅନ� ବିକ�ଗୁଡ଼ିକର ସଂ�ି� ବିବରଣୀ:
o A. ଗଣତା�ିକ ନାଗରି କତା ବିକାଶ: ମା�ମି କ ଶି�ା ଆେୟାଗର ଏହା ଏକ �ମୁଖ ଲ��
ଥ�ଲା। ଗଣତ�େର ଦାୟି �ବାନ ନାଗରି କ େହବା ପାଇ ଁ ଆବଶ�କ ମୂଲ�େବାଧ ଓ ଦ�ତା ବିକାଶ
ପାଇ ଁ ଶି�ା ଛା�ଛା�ୀ�ୁ ସାହାଯ� କରି ବା ଉଚିତ େବାଲି କମି ଶନ ବି�ାସ କରି ଥେ� ଲ।
o C. ଧ�ାମୂଳକ ଦ�ତାର ଉ�ତି: ମା�ମି କ ଶି�ା ଛା�ଛା�ୀ�ୁ ଧ�ାମୂଳକ ଦ�ତା ବିକାଶ ର
ସୁେଯାଗ �ଦାନ କରି ବା ଉଚିତ େବାଲି କମି ଶନ ସୁପାରି ସ କରି ଛ�ି। ଏହା େସମାନ�ୁ େରାଜଗାର
ପାଇ ଁ ��ୁ ତ ◌ି କରି ବାେର ସହାୟକ େହବ ଏବଂ େଦଶର ଅଥ�େନ�ତିକ ବିକାଶେର େଯାଗଦାନ
କରି ବ ।
o D. େନତୃ � ପାଇ ଁ ଶି�ାର ବିକାଶ: େଦଶକୁ ଭବିଷ�ତେର ମାଗ�ଦଶ�ନ କରି ବା ପାଇ ଁ ଆବଶ�କ
େନତୃ � ଗୁଣବିକାଶ ପାଇ ଁ ଶି�ା ଛା�ଛା�ୀ�ୁ ସାହାଯ� କରି ବା ଉଚିତ େବାଲି କମି ଶନ ବି�ାସ
କରି ଥେ� ଲ।

8- The concept of self image and self worth was given by


ଆ� �ତିମ�
ୂ � ଓ ଆ�ମୂଲ�ର ଧାରଣା ଦିଆଯାଇଥ�ଲା

A) Edward Thorndike
B) Abraham Maslow
C) Howard Gardner
D) Carl Rogers
[Odisha CT Exam 2019]

Ans- D

Join Us-
ଓଡ଼ିଶାର ସମ� ପୂବର୍ ବଷର୍ର �ଶ� ଉ�ର ପାଇଁ WtsApp/Call- 8596976190
Download “Tech Of World” App

• Short Explanation: Carl Rogers was a humanistic psychologist who developed


the theory of self-concept. He believed that the self-concept is the individual's
perception of themselves and that it is essential for healthy psychological
development.
• Short Explanation of Other Options:
o A. Edward Thorndike: Thorndike was a psychologist who developed the
connectionist theory of learning.
o B. Abraham Maslow: Maslow was a psychologist who developed the
theory of the hierarchy of needs.
o C. Howard Gardner: Gardner is a psychologist who developed the theory
of multiple intelligences.
• ସଂ�ି� ବିବରଣୀ: କାଲ� େରାଜସ� ଜେଣ ମାନବବାଦୀ ମେନାବି�ାନୀ ଥ�େଲ ଯି ଏ ଆ�-ଧାରଣାର ସି �ା�
ବିକଶିତ କରି ଥେ� ଲ । େସ ବି�ାସ କରୁଥ�େଲ େଯ ଆ�-ଧାରଣା େହଉଛି ବ��ି ର ନିଜ ବିଷୟେର ଧାରଣା
ଏବଂ ସୁ� ମାନସିକ ବିକାଶ ପାଇ ଁ ଏହା ଜରୁରୀ |
• ଅନ� ବିକ�ଗୁଡ଼ିକର ସଂ�ି� ବିବରଣୀ:
o A. ଏଡଓ�ାଡ� ଥନ�ଡିକ୍: ଥନ�ଡାଇକ୍ ଜେଣ ମେନାବି�ାନୀ ଥ�େଲ ଯି ଏ କି ଶି�ାର ସଂେଯାଗବାଦୀ
ସି�ା� ବିକଶିତ କରି ଥେ� ଲ ।
o B. ଆ�ାହମ ମାେ��ା: ମାେ��ା ଜେଣ ମେନାବି�ାନୀ ଥ�େଲ ଯି ଏ ଆବଶ�କତାର ଶୃ�ଳାର ସି �ା�
ବିକଶିତ କରି ଥେ� ଲ ।
o C. ହାୱାଡ� ଗାଡ�ନର: ଗାଡ�ନର ଜେଣ ମେନାବି�ାନୀ ଯି ଏ ଏକାଧକ
� ବୁ �ିର ସି �ା� ବିକଶିତ
କରି ଥେ� ଲ ।

9- Which of the following options is TRUE with reference to the


tool/technique called 'Force field analysis' applied in a school
environment?
� ୁଲ ପରି େବଶେର �େୟାଗ କରାଯାଉଥ�ବା 'େଫାସ� ଫି�� ଆନାଲି ସିସ'୍ ନାମକ ଟୁ ଲ୍
/େଟ�ି�କ୍ ସ���େର ନି�ଲ
� ି ଖତ
� ବିକ� ଗୁଡ଼ିକ ମ�ରୁ େକଉଟିଁ ସତ�?

A) It is a budget management tool that the schools can use to manage


their infrastructure requirements
B) It is a change management tool used to motivate people towards
change and understand resistance
C) It is a useful time management tool which helps in the analysis of the
school time periods

Join Us-
ଓଡ଼ିଶାର ସମ� ପୂବର୍ ବଷର୍ର �ଶ� ଉ�ର ପାଇଁ WtsApp/Call- 8596976190
Download “Tech Of World” App

D) It is an analysis tool that helps in analysing a student's calibre and


provides feedback for improvement
[Odisha CT Exam 2019]

Ans- B

• Short Explanation: Force field analysis is a change management tool that can be
used in a variety of settings, including schools. It is used to identify the forces
that are driving and resisting change, and to develop strategies for overcoming
the resistance.
• Short Explanation of Other Options:
o A.It is a budget management tool that the schools can use to manage
their infrastructure requirements: This is incorrect. Force field analysis
is not a budget management tool.
o C. It is a useful time management tool which helps in the analysis of
the school time periods: This is incorrect. Force field analysis is not a
time management tool.
o D.It is an analysis tool that helps in analysing a student's calibre and
provides feedback for improvement: This is incorrect. Force field
analysis is not used to analyze student performance.

Join Us-
ଓଡ଼ିଶାର ସମ� ପୂବର୍ ବଷର୍ର �ଶ� ଉ�ର ପାଇଁ WtsApp/Call- 8596976190
Download “Tech Of World” App

• ସଂ�ି� ବିବରଣୀ: େଫାସ� ଫି�� ବିେ�ଷଣ େହଉଛି ଏକ ପରି ବ��ନ ପରି ଚାଳନା ଉପକରଣ ଯାହା ବିଦ�ାଳୟ
ସେମତ ବିଭି� େସଟିଂେର ବ�ବହୃ ତ େହାଇପାେର । ଏହା ପରି ବ��ନକୁ ଚଳାଇଥ�ବା ଏବଂ �ତିେରାଧ
କରୁଥ�ବା ଶ�ି ଗଡ଼ ୁ ିକୁ ଚି�ଟ କରି ବା ଏବଂ �ତିେରାଧକୁ ଅତି�ମ କରି ବା ପାଇ ଁ ରଣନୀତି ବିକଶିତ କରି ବା ପାଇ ଁ
ବ�ବହୃ ତ ହୁ ଏ |
• ଅନ� ବିକ�ଗୁଡ଼ିକର ସଂ�ି� ବିବରଣୀ:
o A. ଏହା ଏକ ବେଜଟ୍ ପରି ଚାଳନା ଉପକରଣ ଯାହାକୁ ବିଦ�ାଳୟଗୁଡ଼ିକ େସମାନ�ର ଭି �ିଭୂମି
ଆବଶ�କତା ପରି ଚାଳନା କରି ବା ପାଇ ଁ ବ�ବହାର କରି ପାରି େବ: ଏହା ଭୁଲ୍ । େଫାସ� ଫି��
ବିେ�ଷଣ ଏକ ବେଜଟ୍ ପରି ଚାଳନା ଉପକରଣ ନୁ େହ ଁ।
o C. ଏହା ଏକ ଉପେଯାଗୀ ସମୟ ପରି ଚାଳନା ଉପକରଣ ଯାହା ବିଦ�ାଳୟ ସମୟ ଅବଧ�ର
ବିେ�ଷଣେର ସାହାଯ� କେର: ଏହା ଭୁଲ ଅେଟ । େଫାସ� ଫି�� ବିେ�ଷଣ ଏକ ସମୟ ପରି ଚାଳନା
ଉପକରଣ ନୁ େହ ଁ।
o D. ଏହା ଏକ ବିେ�ଷଣ ଉପକରଣ ଯାହା ଜେଣ ଛା�� ଦ�ତା ବିେ�ଷଣ କରି ବାେର
ସାହାଯ� କେର ଏବଂ ଉ�ତି ପାଇ ଁ ମତାମତ �ଦାନ କେର: ଏହା ଭୁଲ୍ ଅେଟ। ଛା�� �ଦଶ�ନ
ବିେ�ଷଣ କରି ବା ପାଇ ଁ େଫାସ� ଫି�� ବିେ�ଷଣ ବ�ବହୃ ତ ହୁ ଏ ନାହିଁ।

10- Which among the following options is a demerit of


'Demonstration method of learning'?
ନି�ଲ
� ି ଖତ
� ବିକ� ଗୁଡ଼ିକ ମ�ରୁ େକଉଟିଁ 'ଶି�ଣର �ଦଶ�ନ ପ�ତି'ର ଅପମାନ?

A) It helps in arousing the spirit of discovery among students


B) It accounts for the principles of reflective thinking
C) It is a costly method as expensive materials are required
D) It leads to permanent learning
[Odisha CT Exam 2019]

Ans- C

Join Us-
ଓଡ଼ିଶାର ସମ� ପୂବର୍ ବଷର୍ର �ଶ� ଉ�ର ପାଇଁ WtsApp/Call- 8596976190
Download “Tech Of World” App

• Short Explanation: The demonstration method of learning can be


costly, especially when it is used to teach complex concepts that require
expensive materials, such as scientific equipment or art supplies.
• Short Explanation of Other Options:
o A. It helps in arousing the spirit of discovery among students: The
demonstration method can help to arouse the spirit of discovery among
students by showing them new things and inspiring them to ask questions.
o B. It accounts for the principles of reflective thinking: The demonstration
method can help students to develop reflective thinking skills by
encouraging them to think about what they are seeing and to ask
questions about the process.
o D. It leads to permanent learning: The demonstration method can lead to
permanent learning, especially when it is used in conjunction with other
teaching methods, such as discussion and hands-on activities.

• ସଂ�ି� ବିବରଣୀ: ଶି�ଣର �ଦଶ�ନ ପ�ତି ମହ�ା େହାଇପାେର, ବିେଶଷକରି େଯେତେବେଳ ଏହା ଜଟିଳ
ଧାରଣା ଶିଖାଇବା ପାଇ ଁ ବ�ବହୃ ତ ହୁ ଏ େଯଉଥ�
ଁ ପାଇ ଁ େବ��ାନିକ ଉପକରଣ କି�ା� କଳା େଯାଗାଣ ପରି
ବ�ୟବହୁ ଳ ସାମ�ୀ ଆବଶ�କ ହୁ ଏ ।
• ଅନ� ବିକ�ଗୁଡ଼ିକର ସଂ�ି� ବିବରଣୀ:
o A. ଏହା ଛା�ଛା�ୀ� ମ�େର ଆବି�ାରର ଭାବନା ଜା�ତ କରି ବାେର ସାହାଯ� କରି ଥାଏ:
�ଦଶ�ନ ପ�ତି ଛା�ଛା�ୀମାନ�ୁ ନୂ ତନ ଜିନିଷ େଦଖାଇ ଏବଂ ��� ପଚାରି ବାକୁ େ�ରଣା େଦଇ
େସମାନ� ମ�େର ଆବି�ାରର ଭାବନା ଜା�ତ କରି ବାେର ସାହାଯ� କରି ପାେର |
o B. ଏହା �ତିଫଳିତ ଚି�ାଧାରାର ସି �ା�କୁ ହିସାବ କେର: �ଦଶ�ନ ପ�ତି ଛା�ଛା�ୀମାନ�ୁ
େସମାେନ ଯାହା େଦଖୁଛ�ି େସ ବିଷୟେର ଚି�ା କରି ବାକୁ ଏବଂ ��ିୟା ବିଷୟେର ��� ପଚାରି ବା
ପାଇ ଁ ଉ�ାହିତ କରି �ତିଫଳିତ ଚି�ା େକୗଶଳ ବିକଶିତ କରି ବାେର ସାହାଯ� କରି ପାେର |
o D. ଏହା �ାୟୀ ଶି�ଣ କୁ ଆେଗଇ େନଇଥାଏ: �ଦଶ�ନ ପ�ତି �ାୟୀ ଶି�ଣର କାରଣ େହାଇପାେର,
ବିେଶଷକରି େଯେତେବେଳ ଏହାକୁ ଅନ� ଶି�ାଦାନ ପ�ତି, େଯପରି କି ଆେଲାଚନା ଏବଂ ହ�ଗତ
କାଯ��କଳାପ ସହିତ ମିଶାଇ ବ�ବହାର କରାଯାଏ ।

Join Us-
ଓଡ଼ିଶାର ସମ� ପୂବର୍ ବଷର୍ର �ଶ� ଉ�ର ପାଇଁ WtsApp/Call- 8596976190
Download “Tech Of World” App

11- "Micro teaching is a scaled down teaching encountering class


size and class time," said
"ମାଇେ�ା ଟିଚିଂ େହଉଛି େ�ଣୀ ଆକାର ଏବଂ େ�ଣୀ ସମୟ କୁ ଦୃ �ିେର ରଖ � ଶି�ାଦାନ ଏକ
େ�ଲ୍ ଡିଂ" େବାଲି େସ କହିଛ�ି।

A) Griffiths
B) Shore
C) Allen
D) Jangira
[Odisha B.Ed Exam 2018]

Ans- C

• Short Explanation: Allen (1963) is credited with developing the micro teaching
technique.
• Short Explanation of Other Options:
o A. Griffiths: Griffiths is a well-known educational researcher who has written
extensively on the topic of teaching and learning.
o B. Shore: Shore is a professor of education who has developed a model of
classroom management called the "responsive classroom."
o D. Jangira: Jangira is an Indian educationist who has written extensively on
the topic of teacher education.
• ସଂ�ି� ବିବରଣୀ: ଆେଲନ୍ (୧୯୬୩)�ୁ ମାଇେ�ା ଟିଚଂି େକୗଶଳ ବିକଶିତ କରି ବାର େ�ୟ ଦିଆଯାଏ ।
• ଅନ� ବିକ�ଗୁଡ଼ିକର ସଂ�ି� ବିବରଣୀ:
o A. �ି ଫି��: �ିଫି�� ଜେଣ ଜଣାଶୁଣା ଶି�ା ଗେବଷକ ଯି ଏ କି ଶି�ାଦାନ ଏବଂ ଶି�ଣ ବିଷୟେର ବି�ୃତ
ଭାବେର େଲଖ�ଛ�ି ।
o B. େସାର: େସାର୍ ଜେଣ ଶି�ା ଅ�ାପକ ଯିଏ କି େ�ଣୀଗୃହ ପରି ଚାଳନାର ଏକ ମେଡଲ ବିକଶିତ
କରି ଛ�ି ଯାହାକୁ "�ତି�ିୟାଶୀଳ େ�ଣୀଗୃହ" କୁହାଯାଏ ।
o D ଜା�ିରା: ଜ�ିରା ଜେଣ ଭାରତୀୟ ଶି�ାବିତ୍ ଯି ଏ କି ଶି�କ ଶି�ା ବିଷୟେର ବି�ୃତ ଭାବେର
େଲଖ�ଛ�ି ।

Join Us-
ଓଡ଼ିଶାର ସମ� ପୂବର୍ ବଷର୍ର �ଶ� ଉ�ର ପାଇଁ WtsApp/Call- 8596976190
Download “Tech Of World” App

12- At the preschool level, which of the following is a very popular


methodology of imparting education to children?
�ାଥମିକ �ରେର, ନି�ଲ
� ି ଖତ
� ମ�ରୁ େକଉଟିଁ ପିଲାମାନ�ୁ ଶି�ା �ଦାନ କରି ବାର ଏକ
ଅତ�� େଲାକ�ିୟ ପ�ତି?

A) Demonstration
B) Self learning method
C) The play-way method
D) The lecture method
[Odisha B.Ed Exam 2018]

Ans- C

• Short Explanation: The play-way method is a popular methodology of imparting


education to children at the preschool level. It is based on the principle that
children learn best through play. The play-way method involves using a variety
of activities, such as games, songs, and stories, to teach children about the
world around them.
• Short Explanation of Other Options:
o A. Demonstration: The demonstration method can be used to teach
children at the preschool level, but it is not as widely used as the play-way
method.
o B. Self learning method: The self-learning method is not typically used
at the preschool level, as children are still developing the skills needed to
learn independently.
o D. The lecture method: The lecture method is not typically used at the
preschool level, as children have short attention spans and need to learn
through activities.

Join Us-
ଓଡ଼ିଶାର ସମ� ପୂବର୍ ବଷର୍ର �ଶ� ଉ�ର ପାଇଁ WtsApp/Call- 8596976190
Download “Tech Of World” App

• ସଂ�ି� ବିବରଣୀ: େ��-େୱ ପ�ତି େହଉଛି �ାଥମି କ �ରେର ପି ଲାମାନ�ୁ ଶି�ା �ଦାନ କରି ବାର ଏକ
େଲାକ�ିୟ ପ�ତି | ଏହା ଏହି ନୀତି ଉପେର ଆଧାରି ତ େଯ ପି ଲାମାେନ େଖଳ ମା�ମେର େ��� ଶିଖ�ି
। ପି ଲାମାନ�ୁ େସମାନ� ଆଖପାଖ ଦୁ ନଆ ି ବିଷୟେର ଶି�ା େଦବା ପାଇ ଁ େଖଳ, ଗୀତ ଏବଂ କାହାଣୀ ପରି
ବିଭି� କାଯ��କଳାପ ବ�ବହାର କରି ବା ଏହି େ��-େୱ ପ�ତିେର ଅ�ଭ�ୁ� ।
• ଅନ� ବିକ�ଗୁଡ଼ିକର ସଂ�ି� ବିବରଣୀ:
o A. �ଦଶ�ନ: �ାଥମି କ �ରେର ପି ଲାମାନ�ୁ ଶି�ା େଦବା ପାଇ ଁ �ଦଶ�ନ ପ�ତି ବ�ବହାର
କରାଯାଇପାେର, କି� ଏହା େ��-େୱ ପ�ତି ପରି ବ�ାପକ ଭାବେର ବ�ବହୃ ତ ହୁ ଏ ନାହିଁ ।
o B. ଆ�ଶି�ଣ ପ�ତି: ଆ�-ଶି�ଣ ପ�ତି ସାଧାରଣତଃ �ାଥମିକ �ରେର ବ�ବହୃ ତ ହୁ ଏ ନାହିଁ,
କାରଣ ପି ଲାମାେନ �ାଧୀନ ଭାବେର ଶିଖବ
� ା ପାଇ ଁ ଆବଶ�କ ଦ�ତା ବିକଶିତ କରୁଛ�ି ।
o D. ବକ୍ତୃତା ପ�ତି: ବକ୍ତୃତା ପ�ତି ସାଧାରଣତଃ �ାଥମି କ �ରେର ବ�ବହୃ ତ ହୁ ଏ ନାହି,ଁ କାରଣ
ପିଲାମାନ�ର �ାନ ଅବଧ � କମ୍ ଥାଏ ଏବଂ କାଯ��କଳାପ ମା�ମେର ଶିଖବ
� ାର ଆବଶ�କତା ଥାଏ ।

13- When education is acquired without any specific purpose, fixed


period and place, it is known as
େକୗଣସି ନି��� ଉେ�ଶ�, ନି��� ଅବଧ � ଓ �ାନ ବିନା ଶି�ା �ା� େହେଲ ଏହାକୁ କୁହାଯାଏ

A) Independent Education
B) Informal education
C) Formal education
D) Direct education
[Odisha B.Ed Exam 2018]

Ans- B

Join Us-
ଓଡ଼ିଶାର ସମ� ପୂବର୍ ବଷର୍ର �ଶ� ଉ�ର ପାଇଁ WtsApp/Call- 8596976190
Download “Tech Of World” App

• Short Explanation: Informal education is the type of education that is acquired


without any specific purpose, fixed period, and place. It is the learning that takes
place through everyday experiences, interactions, and observations. For
example, a child learns about the world around them by watching their parents
and siblings, playing with friends, and exploring their environment.
• Short Explanation of Other Options:
o A. Independent education is a type of education in which students learn
independently, without the guidance of a teacher.
o C. Formal education is the type of education that is provided in a structured
setting, such as a school or university.
o D. Direct education is a type of education in which the teacher directly
instructs the students.

• ସଂ�ି� ବିବରଣୀ: ଅେନୗପଚାରି କ ଶି�ା େହଉଛି ଏକ �କାର ଶି�ା ଯାହା େକୗଣସି ନି��� ଉେ�ଶ�,
ନି�ା� ରି ତ ଅବଧ � ଏବଂ �ାନ ବିନା ଅଧ�
� ହଣ କରାଯାଏ । େଦ�ନ�ିନ ଅନୁ ଭୂତି, ବା��ାଳାପ ଏବଂ ପଯ��େବ�ଣ

ମା�ମେର ଏହି ଶି�ା ହି େହାଇଥାଏ । ଉଦାହରଣ �ରୂପ, ଜେଣ ପି ଲା ନିଜ ପି ତାମାତା ଏବଂ ଭାଇଭଉଣୀ�ୁ
େଦଖ�, ସା�ମାନ� ସହିତ େଖଳିବା ଏବଂ େସମାନ� ପରି େବଶ ଅନୁ ସ�ାନ କରି େସମାନ� ଆଖପାଖର
ଦୁ ନିଆ ବିଷୟେର ଶିଖଥ � ାଏ ।
• ଅନ� ବିକ�ଗୁଡ଼ିକର ସଂ�ି� ବିବରଣୀ:
o A. �ାଧୀନ ଶି�ା େହଉଛି ଏକ �କାରଶି�ା େଯଉଥ�
ଁ େର ଛା�ମାେନ ଶି�କ� ମାଗ�ଦଶ�ନ ବିନା
�ାଧୀନ ଭାବେର ଶିଖ�ି ।
o C. ଆନୁ �ା� ନିକ ଶି�ା େହଉଛି ଏକ �କାରଶି�ା ଯାହା ଏକ ବିଦ�ାଳୟ କି�ା� ବି�ବିଦ�ାଳୟ ପରି ଏକ
ଗଠନମୂଳକ େସଟିଂେର �ଦାନ କରାଯାଏ ।
o D. �ତ�� ଶି�ା ଏକ �କାର ଶି�ା େଯଉଥ�
ଁ େର ଶି�କ ସି ଧାସଳଖ ଛା�ଛା�ୀ�ୁ ନିେ��ଶ ଦିଅ�ି ।

Join Us-
ଓଡ଼ିଶାର ସମ� ପୂବର୍ ବଷର୍ର �ଶ� ଉ�ର ପାଇଁ WtsApp/Call- 8596976190
Download “Tech Of World” App

14- In India, there are various bodies governing school education


system such as Islamic Madrasah schools, whose boards are
controlled by local state governments, or autonomous, or affiliated
with
ଭାରତେର, ବିଦ�ାଳୟ ଶି�ା ବ�ବ�ାକୁ ନିୟ�ଣ କରୁଥ �ବା ବିଭି� ସଂ�ା ଯଥା ଇ��ାମି କ ମ�ାସା
ବିଦ�ାଳୟ ଅଛି , ଯାହାର େବାଡ�ଗଡ ୁ କ
ି �ାନୀୟ ରାଜ� ସରକାର� �ାରା ନିୟ�ିତ କି�ା�
�ୟଂଶାସି ତ କି�ା� ଅନୁ ବ�ିତ ଅେଟ ।

A) Al-Maqar, Darul Aman


B) Al-Azhar
C) DarulUloom Deoband.
D) Jamia Nizamia
[Odisha B.Ed Exam 2018]

Ans- C
• Short Explanation: DarulUloom Deoband is an Islamic seminary located in
Deoband, India. It is one of the largest and most influential Islamic seminaries in
the world. DarulUloom Deoband is an autonomous institution and is not affiliated
with any government.
• Short Explanation of Other Options:
o A. Al-Maqar and Darul Aman are institutions located in Saudi Arabia.
o B. Al-Azhar is a university located in Egypt.
o D. Jamia Nizamia is an institution located in Hyderabad, India.
• ଦାରୁଲୁ ଲୁମ ୍େଦଓବ� ଭାରତର େଦଓବ�େର ଅବ�ି ତ ଏକ ଇସଲାମି କ ମ�ାସା । ଏହା ବି�ର ସବୁ ଠାରୁ
ବଡ଼ ଏବଂ �ଭାବଶାଳୀ ଇସଲାମିକ ମ�ାସା ମ�ରୁ ଅନ�ତମ । ଦାରୁଲୁ ମ େଦଓବ� ଏକ �ୟଂଶାସି ତ
ଅନୁ �ା� ନ ଏବଂ ଏହା େକୗଣସି ସରକାର� ସହ ଅନୁ ବ�ିତ ନୁ େହ ଁ।
• ଅନ� ବିକ�ଗୁଡ଼ିକର ସଂ�ି� ବିବରଣୀ:
o A. ଅଲ୍ -ମକାର ଏବଂ ଦାରୁଲ୍ ଅମାନ େହଉଛି ସାଉଦି ଆରବେର ଅବ�ି ତ ଅନୁ �ା� ନ ।
o B. ଅଲ ଆଝାର ଇଜି�େ� ର ଅବ�ି ତ ଏକ ବି�ବିଦ�ାଳୟ ।
o C. ଜାମି ଆ ନିଜାମି ଆ ଭାରତର ହାଇ�ାବାଦେର ଅବ�ି ତ ଏକ ଅନୁ �ା� ନ ।

To Continue- Click Here

Join Us-
ଓଡ଼ିଶାର ସମ� ପୂବର୍ ବଷର୍ର �ଶ� ଉ�ର ପାଇଁ
WtsApp/Call- 8596976190
Download “Tech Of World” App

ALL ODISHA REASONING PYQ


E- Book (PDF)
All Odisha Exam Previous Year Chapter Wise
Reasoning Question
(Odia & English Language)
[Exam- OSSC, OSSSC, Police, B.ED, RHT, CT, JT, OAVS, OSSTET,
Battalion, Fireman & Other Exams]

Tech Of World APP

Features Of “REASONING” Chapter Wise E-Book


Total PYQ 4,181

Total Topic 21 (With Answer Key)

Language Odia & English

Best For All Odisha Exam- [Exam- OSSC, OSSSC, Police, B.ED,
RHT, CT, JT, OSSTET, Battalion, Fireman & Other Exams]

Download App To Buy Click Here

Contact 8596976190

Click Here

Join Us-
ଓଡ଼ିଶାର ସମ� ପୂବର୍ ବଷର୍ର �ଶ� ଉ�ର ପାଇଁ
WtsApp/Call- 8596976190
Download “Tech Of World” App

Sl. No. Topic No. Of MCQ

1 Coding-Decoding 590
2 Number & Alphabet Series 569
3 Statement And Conclusion 432
4 Blood Relations 372
5 Seating Arrangement 284
6 Statements 161
7 Ranking And Ordering 167
8 Analogy 258
9 Direction and Distance 255
10 Inequality 163
11 Clock, Day, Calendar 91
12 Statement & Argument 40
13 Figure Series Test 159
14 Miscellaneous 162
15 Missing Number Quiz 26
16 Odd Man Out 203
17 Statement & Assumption 80
18 Water & Mirror Image 50
19 Arithmetic Reasoning 63
20 Arranging Word 41
21 Cube & Dice 15
Total 4,181

Join Us-
ଓଡ଼ିଶାର ସମ� ପୂବର୍ ବଷର୍ର �ଶ� ଉ�ର ପାଇଁ
WtsApp/Call- 8596976190
Download “Tech Of World” App

Topic Name- Blood Relation

1- How is my father's sister's son's sister related to me?

- େମା ବାପା� ଭଉଣୀର ପୁଅର ଭଉଣୀ େମା ସହିତ କିପରି ସ�ୃ�?

A. Grandmother
B. Cousin
C. Data inadequate
D. None of these
[Odisha Jail Warder 2022]

Ans- B

2- Pointing to a girl, a woman said, "She is my mother's only


daughter's husband's son-in-law's daughter's only sister." How is
the woman related to the girl?

- ଜେଣ ଝିଅକୁ େଦଖାଇ ଜେଣ ମହିଳା କହିେଲ, େସ େମା ମା'ର ଏକମା� ଝିଅର �ାମୀ� �ାଇଁ� ଝିଅର
ଏକମା� ଭଉଣୀ। ମହିଳା ଜଣକ େକମିତ ି ଝିଅ ସହ ସ�ୃ�?

(A) Mother
(B) Grandmother
(C) Daughter
(D) Aunt
[OSSC BSSO 2022]

Ans- B

Join Us-
ଓଡ଼ିଶାର ସମ� ପୂବର୍ ବଷର୍ର �ଶ� ଉ�ର ପାଇଁ
WtsApp/Call- 8596976190
Download “Tech Of World” App

3- Pointing to a woman, a girl said, "She is the mother of niece of


my sister's only maternal aunt who has no brother." How is the girl
related to the woman?

- ଜେଣ ମହିଳା� ଆଡ଼କୁ ଇ�ି ତ କରି ଜେଣ ଝିଅ କହିଲା, େସ େମା ଭଉଣୀର ଏକମା� ମାଉସୀ�
ଝିଆରୀର ମା' ଯାହାର େକୗଣସି ଭାଇ ନାହିଁ। ମହିଳା� ସହ ଝିଅର ସ�କର୍ େକମିତ?
ି

(A) Sister
(B) Granddaughter
(C) Daughter
(D) Niece
[OSSC CGL Exam 2022]

Ans- C

4- Kapil is the brother of Nikhil and Sunil, Yamuna is the mother of


Nikhil and Jawahar is the father of Kapil. Which one of the following
relations is FALSE?

କପିଲ ନିଖିଲ ଓ ସୁନୀଲ� ଭାଇ, ଯମୁନା ନିଖିଲ� ମା' ଏବଂ ଜବାହର କପିଲ� ବାପା । ନିମଲ
� ିଖିତ
ସ�କର୍ ଗୁଡ଼କ
ି ମଧ୍ୟରୁ େକଉଁଟ ି ଭୁଲ୍ ଅେଟ?

A) Kapil is the son of Yamuna


B) Yamuna is the wife of Jawahar
C) Kapil is the son of Jawahar
D) Kapil is the father of Sunil
[OSSC Junior Assistant 2022 Exam]

Ans- D

Join Us-
ଓଡ଼ିଶାର ସମ� ପୂବର୍ ବଷର୍ର �ଶ� ଉ�ର ପାଇଁ
WtsApp/Call- 8596976190
Download “Tech Of World” App

5- Looking at a portrait of a man, Harsh said, “His mother is the


wife of my father’s son. Brothers and sisters I have none.” At
whose portrait was Harsh looking?

- ଜେଣ ବ୍ୟ�ି� ଫେଟା େଦଖି ହଷର୍ କହିେଲ, 'ତା� ମା' େହଉଛ�ି େମା ବାପା� ପୁଅର ପ�ୀ। ଭାଇ ଓ
ଭଉଣୀମାେନ େମାର େକହି ନାହା�ି। ହଷର୍ କାହା ର ଫେଟା େଦଖୁଥିେଲ ?

A- His son
B- His cousin
C- His uncle
D- His nephew
[Odisha Police SI 2022 Exam]

Ans- A

6- Pointing to a lady in the photograph, Seema said, "Her


daughter's son's grandmother is my mother." How may Seema be
related to the lady's grandson?

- ଫେଟାେର ଥିବା ଜେଣ ମହିଳା�ୁ ଇ�ି ତ କରି ସୀମା କହିଥିେଲ, ତା� ଝିଅର ପୁଅର େଜେଜମା'
େହଉଛ�ି େମା ମା' । ମହିଳା� ନାତି� ସହ ସୀମା� ସ�କର୍ େକମିତ ି େହାଇପାେର?

A) Daughter
B) Niece
C) Grandmother
D) Aunt
[Odisha High School Teacher Exam 2021]

Ans- A

Join Us-
ଓଡ଼ିଶାର ସମ� ପୂବର୍ ବଷର୍ର �ଶ� ଉ�ର ପାଇଁ
WtsApp/Call- 8596976190
Download “Tech Of World” App

7- Pointing to a man in a photograph, woman says, "he is the father


of my only daughter in law's father-in-law." How is the man related
to the woman?

- ଫେଟାେର ଥିବା ଜେଣ ପୁରୁଷ�ୁ େଦଖାଇ ମହିଳା ଜଣକ କହ�ି, େସ େମା ର ଏକମା� େବାହୂ ର
�ଶୁର� ବାପା। ପୁରୁଷ� ସ�କର୍ ମହିଳା� ସହ େକମିତ?ି

(A) Brother
(B) Husband
(C) Father-in-law
(D) Father
[Odisha B.Ed Exam 2022]

Ans- C

8- Rakesh is the son-in-law of Jeevan's wife's father. How is


Jeevan's wife related to Rakesh's wife?

- ରାେକଶ େହଉଛ�ି ଜୀବନ� ପ�ୀ� ବାପା� �ାଇଁ। ଜୀବନ� ପ�ୀ ରାେକଶ� ପ�ୀ� ସହ
େକମିତ ି ସ�କର୍ ରଖିଛ�ି?

A) Sister
B) Daughter
C) Niece
D) Mother
[OSSC Traffic Constable 2022 Exam]

Ans- A

Join Us-
ଓଡ଼ିଶାର ସମ� ପୂବର୍ ବଷର୍ର �ଶ� ଉ�ର ପାଇଁ
WtsApp/Call- 8596976190
Download “Tech Of World” App

9- Pointing to a photograph of a girl, a man said "She is the


paternal grandmother of daughter of brother of my sister". How is
the girl related to the man?

- ଜେଣ ଝିଅର ଫେଟା େଦଖାଇ ଜେଣ ବ୍ୟ�ି କହିେଲ "େସ େମା ଭଉଣୀର ଭାଇ� ଝିଅର େଜେଜମା"।
ଯୁବତୀ� ସ�କର୍ ପୁରୁଷ� ସହ େକମିତ?
ି

(A) Niece
(B) Mother
(C) Sister
(D) Sister-in-law
[Odisha Police ASI 2022]

Ans- B

10- Pointing to a photograph, a man said, "I have no brother or


sister but that man's father is my father's son".. Whose photograph
was it?

- ଜେଣ ବ୍ୟ�ି ଏକ ଫେଟା େଦଖାଇ କହିେଲ, "େମାର େକୗଣସି ଭାଇ କିମ�ା ଭଉଣୀ ନାହିଁ କି�ୁ େସହି
ବ୍ୟ�ି� ପିତା େମା ବାପା� ପୁଅ". ଏହା କାହାର ଫେଟା ଥିଲା?

(A) His own


(B) His nephew's
(C) His father's
(D) His son's
[Odisha OPSC OMAS Exam 2019]

Ans- D

Join Us-
ଓଡ଼ିଶାର ସମ� ପୂବର୍ ବଷର୍ର �ଶ� ଉ�ର ପାଇଁ
WtsApp/Call- 8596976190
Download “Tech Of World” App

11- When Manoj saw Raju, he recalled, ಯHe is the son of the father
of the mother of my daughter.ರHow is Manoj related to Raju?

- ମେନାଜ େଯେତେବେଳ ରାଜୁ �ୁ େଦଖିେଲ, େସ ମେନ ପକାଇେଲ, "େସ େମା ଝିଅର ମା'� ପିତା�
ପୁଅ | ମେନାଜ ରାଜୁ � ସହ େକମିତ ି ସ�ୃ� ?

A) Brother-in-law
B) Father-in-law
C) Son-in-law
D) Grandfather
[OSSC Junior Assistant 2022 Exam]

Ans- A

12- John is the father of Madan. Madan's sister is Teena. How is


John related to Teena?

- େଯାହନ େହଉଛ�ି ମଦନ� ପିତା । ମଦନ� ଭଉଣୀ େହଉଛ�ି ଟିନା। ଜନ୍ ଟିନା� ସହ କିପରି
ସ�ୃ�?

A) Son
B) Cousin
C) Father
D) Brother in law
[Odisha CT Exam 2018]

Ans- C

Join Us-
ଓଡ଼ିଶାର ସମ� ପୂବର୍ ବଷର୍ର �ଶ� ଉ�ର ପାଇଁ
WtsApp/Call- 8596976190
Download “Tech Of World” App

To Continue- Click Here

Join Us-

You might also like